Você está na página 1de 39

1 Números Finitos e Infinitos

Neste capı́tulo, será estabelecida com precisão a diferença entre conjunto finito e con-
junto infinito. Será feita também a distinção entre conjunto enumerável e conjunto não-
enumerável. O ponto de partida é o conjunto dos números naturais.

1.1 Números Naturais


Os números naturais N são construı́dos pelos 3, seguintes, axiomas de Peano:

1. Existe uma função injetiva s: N −→ N. A imagem s(n) de cada número natural


n ∈ N chama-se o sucessor de n.

2. Existe um único número natural 1 ∈ N tal que 1 6= s(n) para todo n ∈ N.

3. Se um conjunto X ⊂ N é tal que 1 ∈ X e s(X) ⊂ X (isto é, n ∈ X ⇒ s(n) ∈ X)


então X = N.

O axioma 3 é conhecido como o princı́pio da indução.


O princı́pio da indução serve de base para um método de demonstração de teoremas
sobre número naturais, conhecido como o

Método de Indução (ou Recorrência) : “se uma propriedade P é válida para o número
1 e se, supondo P válida para o número n > 1 daı́ resultar que P é válida também para
seu sucessor s(n), então P é válida para todos os números naturais”.
Por exemplo, provemos a propriedade seguinte

Propriedade: ∀n ∈ N, tem-se s(n) 6= n.

Demonstração
Esta afirmação é verdadeira para n = 1 porque, pelo axioma 2, tem-se 1 6= s(n) para
todo n ∈ N e, em particular, 1 6= s(1).
Suponhamos que n 6= s(n) é verdadeira para algum natural n > 1. Agora, o axioma
1 nos garante que s é injetiva, aplicando essa injetividade em n 6= s(n) obtemos s(n) 6=
s(s(n)).
Logo, s(n) 6= n é verdadeira ∀n ∈ N. 

Se n, m ∈ N temos que n + m ∈ N (adição) e n · m ∈ N (multiplicação) implicando as


seguinte propriedades:

P1) m +1= s(m);

P2) m + s(n) = s(m + n), isto é, m + (n +1) = (m + n) +1;

P3) m · 1 = m;

P4) m · (n + 1) = m · n + m;

P5) Associatividade: (m + n) + p = m + (n + p), m · (n · p) = (m · n) · p;

P6) Distributividade: m · (n + p) = m · n + m · p;

1
P7) Comutatividade: m + n = n + m, m · n = n · m;

P8) Lei do corte: (Soma) m + n = m + p ⇒ n = p, (Produto) m · n = m · p ⇒ n · p.

Observação: O número 0 (zero) pelos axiomas de Peano não pertence ao conjunto dos
números naturais, pois 1 é o menor elemento nessa construção, daı́ podemos fazer a lei
do corte no produto acima sem restrições.
Dados m, n ∈ N, diz-se que m é menor que n, em sı́mbolos fica m < n quando existe
p ∈ N tal que n = m + p. A notação m ≤ n significa que m < n ou m = n.

(Transitividade) se m, n, p ∈ N com m < n e n < p então m < p.

Demonstração
De fato, como m < n ⇒ ( ∃r ∈ N; n = m + r e como n < p ⇒ ∃s ∈ N; p = n + s
n= m+r
Encontrando assim o sistema , o que implica
p= n+s

p = (m + r) + s ⇒ p = m + (r + s) ⇒ m < p. 

Exemplo ∀n ∈ N, @a ∈ N; n < a < n + 1

Demonstração
Suponha, por absurdo, que exista um n ∈ N; n < a < n + 1, com a ∈ N, isso implica
que ∃p ∈ N tal que a = n + p. Da mesma forma, ∃q ∈ N tal que n + 1 = a + q. Daı́
encontramos
(
a= n+p
n+1= a+q

n+a+1=a+n+p+q

1=p+q

p<1eq<1
(absurdo, pois viola o axioma 2) 

Temos, também, que dados m, n ∈ N quaisquer, vale uma, e somente uma, das três
alternativas: m = n, m < n ou n < m, esta última é chamada propriedade da tricotomia.
Para finalizarmos esse capı́tulo de números naturais façamos a demonstração do

Princı́pio da Boa Ordenação : Todo subconjunto não vazio A ⊂ N possui um menor


elemento, isto é, um elemento n0 ∈ A é tal que n0 < n para todo n ∈ A.

Demonstração
Primeiramente, tomemos, aleatoriamente, um subconjunto não vazio A ⊂ N.

2
Caso 1 ∈ A, pelo axioma 2, temos que 1 é o menor elemento de A.
Caso 1 ∈ / A, definamos um conjunto B = {b; b < a, ∀a ∈ A}, obviamente, 1 ∈ B pois
1 ≤ n, ∀n ∈ N, em particular, 1 < a, ∀a ∈ A. Mas como B 6= N, temos, pelo 3o axioma,
que ∃b0 ∈ B tal que b0 + 1 ∈ / B. Isso implica que ∃a0 ∈ A tal que a0 ≤ b0 + 1 (I). Daı́,
como b0 ∈ B, temos que b0 < a0 (II) e juntando (I) com (II) temos b0 < a0 ≤ b0 + 1. Ou
seja, a0 = b0 + 1 ∈ A. Temos ainda que b0 − 1, b0 − 2, b0 − 3, · · · , 1 ∈ B, consequentemente

/ A. Daı́, b0 + 1 é o menor elemento de A. E como A foi tomado aleatoriamente, o
princı́pio é válido para todo subconjunto não vazio dos N. 

1.2 Conjuntos Finitos


Definamos o conjunto In = {p ∈ N; p < n}, ou seja, In = {1, 2, 3, · · · , n}. Um conjunto X
diz-se finito quando é vazio ou então existem n ∈ N e uma bijeção f : In → X.

Lema: Se existe uma bijeção f : X → Y com a ∈ X, b ∈ Y e f (a) 6= b então, existe


também uma bijeção g : X → Y tal que g(a) = b.

Ilustração

Figure 1: Note que f (a) = c 6= b ⇒ f (a) 6= b

Demonstração
Se X for um conjunto unitário então, como f é bijeção, Y , também, é um conjunto
unitário. E o lema não se adapta a esse caso.
Consideremos X com 2 elementos ou mais. Daı́ Tome a ∈ X, chame f (a) = c, como
X possui 2 ou mais elementos e é bijeção, então ∃b ∈ Y , com b 6= c implicando f (m) = b
e pela injetividade de f temos m 6= a .
Construı́mos a função g : X → Y tal que g(a) = b, g(m) = c e f (x) = g(x),
∀x 6= a e x 6= m.
Note que a 6= m ⇒ g(a) = b 6= c = g(m) ⇒ g(a) 6= g(m),ou seja, g é injetiva
nessa parte. Além disso f (x) = g(x) ∀x 6= a e x 6= m, e como f é injetiva, então
g é injetiva nessa outra parte, logo g é injetiva em todo X. Temos, também, que
g(X) = {b} ∪ {c} ∪ f (X)[∀f (x), com x 6= a e x 6= m] = Y ⇒ g(X) = Y , ou seja,

3
g é sobrejetiva. Portanto g é bijeção. 

Teorema 1: Todos os subconjuntos de quaisquer conjuntos finitos são finitos.

Demonstração
Tomemos um conjunto finito X de n elementos com a1 ∈ X.
Pelo lema e como X é finito então existe uma bijeção f : In → X, tal que f (n) = a1 .
Se n = 1 então X − {a1 } = ∅ é finito.
Dado n > 1, a restrição de f a In−1 é uma bijeção sobre X − {a1 }, logo X − {a1 } é
finito e tem n − 1 elementos. Esse processo pode ser feito para quaisquer ai ∈ X.
Analogamente, tome um a2 ∈ X com a2 6= a1 , pelo lema podemos ter também
f (n − 1) = a2 , daı́ fn−2 : In−2 → X − {{a1 }, {a2 }} é bijeção, sendo X − {{a1 }{a2 }}
finito com n − 2 elementos. Esse processo pode ser feito com quaisquer duplas de ai , aj ∈
X com ai 6= aj .
Fazendo isso para as ternas, quadras, etc até os grupos de n elementos encontraremos
por exaustão que todos os subconjuntos de X são finitos.
Visto que tomamos um conjunto X finito aleatório, concluı́mos que todos os subcon-
junto de quaisquer conjuntos finitos são finitos. 

Corolário 1: Dada f : X → Y , se Y é finito e f é injetiva então X é finito; se X é


finito e f é sobrejetiva então Y é finito.

Demonstração
Na primeira afirmação como f é injetiva temos que f (X) ⊆ Y , e Y finito implica, pelo
Teorema 1, f (X) finito, logo ∃g : In → f (X) bijetiva. Temos também que h : X → f (X)
é bijetiva, daı́ h−1 : f (X) → X é bijetiva. Fazendo h−1 ◦ g : In → f (X) → X, temos
h−1 ◦ g : In → X, ou seja, X é finito.
Ilustração para a segunda afirmação

Figure 2: Exemplo de função h ◦ f injetiva

Na segunda afirmação f sendo sobrejetiva implica f (X) = Y o que nos assegura a


definir a função h : Y → X injetiva tal que h(f (x)) = x. Como X é finito, E h é injetiva,
pelo que acabamos de provar na primeira afirmação temos que Y é finito. 

4
Corolário 2: Um subconjunto X ⊂ N é finito se, e somente se, é limitado.

Demonstração
(ida) X é finito ⇒ ∃g : In → X bijeção. Ou seja, X possui m elementos. Tomando
xm =máx {X} sendo o maior elemento dos m elementos de X, temos que x ≤ xm , ∀x ∈
X ⇒ X é limitado.
(volta) X limitado ⇒ ∃ p ∈ N; ∀x ∈ X, tem − se x ≤ p ⇒ X ⊂ Ip . E, pelo que
acabamos de provar no Teorema 1, X é finito. 

Teorema 2: Se A é um subconjunto próprio de In , não pode existir uma bijeção


f : A → In .

Demonstração
Suponhamos, por absurdo, que ∃fi : Ai → Imi sendo bijeções com Ai ( Imi . Ou seja,
existe uma contagem de funções fi que não satisfazem o Teorema 2, mas o P.B.O, nos
assegura que nessa contagem existe uma função, dentre as fi , que possui a quantidade
de elementos em seu domı́nio menor do que a quantidade de elementos dos domı́nios das
outras. Como a contagem dos elementos do domı́nio é estabelecida pelos mi , digamos que
m0 = min{mi } seja a quantidade desse menor domı́nio e que a função seja f0 : A0 → Im0
. A partir daqui temos dois casos a serem analisados,

ou

1o caso: mo ∈ A0 . Daı́, pelo lema, ∃g0 : A0 → Im0 bijeção com g0 (m0 ) = m0 . Fazendo a
restrição de g0 no domı́nio A0 − m0 temos g0 : A0 − m0 → Im0 −1 bijeção. Ou seja, m0 − 1
representa a quantidade de elementos do domı́nio A0 − m0 que não satisfaz o Teorema 2,
mas isso é um absurdo por que a menor quantidade de elementos em um domı́nio que não
satisfaz o teorema 2 é m0 .

ou

2o caso: mo ∈ / A0 . Daı́, como f0 é sobrejetiva, ∃a ∈ Ao tal que f0 (a) = m0 . Fazendo


a restrição de f0 no domı́nio A0 − a temos f0 : A0 − a → Im0 −1 bijeção. Pelo mesmo
argumento no 1o caso, m0 − 1 contraria a minimalidade de m0 .
Logo, o Teorema 2 é verdadeiro. 

Corolário 1: Se f : Im → X e g : In → X são bijeções então m = n.

Demonstração
Suponhamos, por absurdo, que m < n, isso implica que Im ( In , pelo Teorema 2 não
existe bijeção de Im para In . Mas existe, veja
g : In → X bijeção ⇒ g −1 : X → In é bijeção ⇒ g −1 ◦ f : Im → X → In ⇒ g −1 ◦ f :
Im → In é bijeção.
Suponha, novamente por absurdo, mas dessa vez que n < m, isso implica que In ( Im ,
pelo Teorema 2 não existe bijeção de In para Im . Mas existe, basta tomar a inversa de
−1
g −1 ◦ f : Im → In , ou seja, (g −1 ◦ f ) : In → Im .
Logo, pela tricotomia, só nos resta a opção m = n. 

5
Corolário 2: Seja X um conjunto finito. Uma aplicação f : X → X é injetiva se, e
somente se, é sobrejetiva.

Demonstração:
(ida) X finito ⇒ ∃g : In → X bijeção ⇒ g −1 : X → In bijeção.
Suponhamos, por absurdo, que f : X → X seja injetiva, mas não sobrejetiva.
Pelo Teorema 1, f (X) é finito, pois é subconjunto próprio de X que é finito, isto é,
f (X) ( X, o que implica, ∃h : Im → f (X) bijeção. Mas como f não é sobrejetiva, isso
implica que m < n. Mas f : X → f (X) é bijeção, daı́ f −1 : f (X) → X também é bijeção.
Fazendo f −1 ◦ h : Im → f (X) → X ⇒ f −1 ◦ h : Im → X bijeção.
Fazendo g −1 ◦ (f −1 ◦ h) : Im → X → In ⇒ g −1 ◦ (f −1 ◦ h) : Im → In bijeção, o que
contraria o Teorema 2.
(volta) Suponhamos, por absurdo, que f : X → X seja sobrejetiva, mas não injetiva.
Isso implica que ∃x1 , x2 ∈ X (dominio) com x1 6= x2 tal que f (x1 ) = f (x2 ), isso nos
garante que o número de elementos do domı́nio X é maior do que o número de elementos
da imagem f (X) pois x1 e x2 contou duas vezes o elemento f (x1 ) = f (x2 ), e como f (X)
é finito temos que ∃h : Im → f (X) bijeção com m < n, mas como f é sobrejetiva então
f (X) = X, logo h : Im → X bijeção com m < n. Daı́,
g −1 ◦ h : Im → X → In ⇒ g −1 ◦ h : Im → In bijeção, o que novamente contraria o
Teorema 2.
Portanto, o Corolário 2 é válido. 

Corolário 3: Não pode existir uma bijeção entre um conjunto finito e uma parte própria
sua .

Demonstração
Suponhamos, por absurdo, que exista um conjunto finito X, tal que f : X → A é
bijeção com A ( X, isso implica que f −1 : A → X é bijeção .
Como X é finito, temos que ∃h : In → X bijeção implicando que h−1 : X → In é
bijeção .
Temos, também, que A ( X implica que a quantidade de elementos de A é menor do
que a quantidade de elementos de X e A finito, daı́ ∃g : Im → A bijeção com m < n.
Agora, f −1 ◦ g : Im → A → X ⇒ f −1 ◦ g : Im → X é bijeção. E h−1 ◦ (f −1 ◦ g) :
Im → X → In . O que implica que h−1 ◦ (f −1 ◦ g) : Im → In é bijeção, o que contraria o
Teorema 2. 

1.3 Conjuntos infinitos


Diz-se que um conjunto é infinito quando não é finito. Assim, X é infinito quando não é
vazio nem existe, seja qual for n ∈ N , uma bijeção f : In → X .

Exemplo
O Corolário 2 do Teorema 1 diz que um subconjunto X ⊂ N é finito se, e somente se,
é limitado. Uma propriedade equivalente a essa é: um subconjunto X ⊂ N é ilimitado se,
e somente se é infinito. Isso assegura que o conjunto N dos números naturais é infinito.
De fato, temos que N é ilimitado, o que implica N infinito.

6
Teorema 3: Se X é um conjunto infinito, então existe uma aplicação injetiva f : N → X.

Demonstração
Tomemos, aleatoriamente, um x1 ∈ X, como X é infinito temos que X − {x1 } 6= ∅,
daı́ podemos tomar um x2 ∈ X − {x1 }, continuando com esse processo temos x3 ∈ X −
{{x1 }, {x2 }}, etc. De forma geral, ficamos com xn ∈ X − {{x1 }, {x2 }, · · · {xn−1 }}. Agora,
construı́mos a função f : N → X pondo f (n) = xn . Note que ∀m, n ∈ N com m 6= n
temos f (m) 6= f (n), pois, suponto m < n, enquanto f (m) ∈ {{x1 }, {x2 }, · · · {xn−1 }}, o
f (n) ∈ X − {{x1 }, {x2 }, · · · {xn−1 }} . Logo, f é injetiva. 

Corolário: Um conjunto X é infinito se, e somente se, existe uma bijeção φ : X → Y ,


onde Y é algum subconjunto próprio de X.

Demonstração
(ida) Como Y é algum subconjunto próprio do conjunto infinito X podemos dizer
que Y = X − {x1 }, onde x1 ∈ X. Aproveitando a função injetiva f na demonstração
do Teorema 3 podemos fazer φ(xn ) = f (n + 1), daı́ φ(xn ) = xn+1 , agora façamos a
identidade φ(x) = x, ∀x 6= xn (n ∈ N) que é injetiva também, juntando essas duas partes
garantimos que φ é injetiva. Note que nosso domı́nio são os elementos da sequência
{{x1 }, {x2 }, {x3 }, · · · {xn }, · · · } mais os elementos restantes de X que não fazem parte da
sequência, ou seja, nosso domı́nio é o próprio X. E nossa imagem são os elementos da
sequência {{x2 }, {x3 }, · · · {xn }, · · · } mais os elementos restantes de X que não fazem parte
da sequência, exceto o x1 , ou seja, nossa imagem é igual ao contradomı́nio Y = X − {x1 }.
Logo, φ(x) é injetiva e sobrejetiva, portanto é uma bijeção.
(volta) O Corolário 3 do Teorema 2 nos diz que Não pode existir uma bijeção entre
um conjunto finito e uma parte própria sua. Então caso exista essa bijeção o conjunto é
infinito, o que prova a volta. 

Exemplo
Se N1 = N − {1} então φ : N → N1 , φ(n) = n + 1, é uma bijeção de N sobre seu
subconjunto N1 = {2, 3, · · · }.

1.4 Conjuntos enumeráveis


Um conjunto X diz-se enumerável quando é finito ou quando existe uma bijeção f :
N → X. Neste caso, f chama-se uma enumeração dos elementos de X. Escrevendo
f (1) = x1 , f (2) = x2 , ..., f (n) = xn , tem-se então X = {x1 , x2 , · · · , xn , · · · } .

Teorema 4: Todo subconjunto X ⊂ N é enumerável.

Demonstração
Caso X ⊂ N seja finito, por definição, ele é enumerável.
Caso X ⊂ N seja infinito, tetemos enumerar os elementos de X pondo x1 sendo o seu
menor elemento. Daı́, como X é infinito, podemos tomar o menor elemento de X − {x1 },
digamos que seja, x2 . Prosseguindo com essa ideia, peguemos x3 o menor elementos
de X − {{x1 }, {x2 }}, etc, e, xn o menor elementos de X − {{x1 }, {x2 }, · · · , {xn−1 }}.
Para que essa tentativa de enumeração, seja de fato uma enumeração, então a função

7
f : N → X, f (n) = xn , nessa construção, deve ser uma bijeção, onde supostamente X =
{{x1 }, {x2 }, · · · , {xn−1 }, · · · }. Note que, dados m, n ∈ N, com m 6= n, digamos m < n ⇒
f (m) ∈ {{x1 }, {x2 }, · · · , {xn−1 }} e f (n) ∈ X −{{x1 }, {x2 }, · · · , {xn−1 }} ⇒ f (m) 6= f (n),
ou seja, m 6= n ⇒ f (m) 6= f (n), então f é injetiva. Se, absurdamente, f não fosse
sobrejetiva, terı́amos a existência de um xk ∈ X satisfazendo que xk 6= xn , ∀n ∈ N. E
veja que xk não poderia está entre nenhuma dupla xi < xi+1 , ∀i ≥ 1, pois se estivesse,
então estaria na lista da função f . Isso implica que xk é maior que todo elemento do
conjunto natural infinito {{x1 }, {x2 }, · · · , {xn−1 }, · · · }, o que é absurdo, pois o tornaria
limitado, pelas caracterı́sticas dos subconjuntos dos números naturais, se um subconjuntos
dos números naturais é limitado então é finito. Logo, f é sobrejetiva. E f sendo injetiva
e sobrejetiva então ela é bijetiva.
Portanto, de fato, f é bijeção, realmente X = {{x1 }, {x2 }, · · · , {xn−1 }, · · · } e o Teo-
rema 4 é verdadeiro. 

Corolário 1: Seja f : X → Y injetiva. Se Y é enumerável então X também é. Em


particular, todo subconjunto de um conjunto enumerável é enumerável.

Demonstração
Temos que Y enumerável ⇒ ∃g : N → Y bijetiva ⇒ g −1 : Y → N também bijetiva.
Como f é injetiva isso implica que f : X → f (X) é bijetiva e f (X) ⊆ Y . A parte
f (X) ⊆ Y nos assegura que g −1 : f (X) → g −1 (f (X)) é bijetiva com g −1 (f (X)) ⊂ N. Isso
implica que a composta g −1 ◦ f : X → g −1 (f (X)) é uma bijeção. Como g −1 (f (X)) ⊂
N, pelo Teorema 4, ele é enumerável. Daı́, temos uma bijeção de X sobre o conjunto
enumerável g −1 (f (X)), o que implica X enumerável.
No caso particular, Tome B um conjunto enumerável e A ⊂ B. Façamos a bijetiva
h : A → h(A) pondo h(x) = x, ∀x ∈ A, obviamente A = h(A) ⊂ B. Como B é enu-
merável então ∃i : B → N bijetiva, como h(A) ⊂ B, isso implica que i : h(A) → i(h(A)) é
bijetiva, com i(h(A)) ⊂ N. Daı́, i ◦ h : A → i(h(A)) é uma bijeção. E como i(h(A)) ⊂ N,
pelo Teorema 4, i(h(A)) é enumerável, , claramente h sendo bijeção de A sobre o conjunto
enumerável i(h(A)) implica que A também é enumerável. 

Corolário 2: Seja f : X → Y sobrejetiva. Se X é enumerável então Y também é.

Demonstração
A função f : X → Y sobrejetiva nos implica que ∃g : Y → X injetiva com g(f (x)) =
x, ∀f (x) ∈ Y . Como X é enumerável e g é injetiva, pelo Corolário1, Y é enumerável. 

Corolário 3: O produto cartesiano de dois conjuntos enumeráveis é um conjunto enu-


merável.

Demonstração
Digamos que os conjuntos X e Y sejam enumerável. Isso implica que existem funções
f : N → X e g : N → Y bijeções, consequentemente f e g são sobrejetivas. Daı́,
temos que a função h : N × N → X × Y definida por h(m, n) = (f (m), g(n)) é sobre-
jetiva. De fato, como f e g são sobrejetivas isso implica que ∀ (f (m), g(n)) ∈ X × Y
teremos f (m) ∈ X e g(n) ∈ Y ⇒ m ∈ N e n ∈ N ⇒ (m, n) ∈ N × N , ou seja,
∀ (f (m), g(n)) ∈ X × Y, ∃(m, n) ∈ N × N tal que h(m, n) = (f (m), g(n)).

8
Ilustração da função h

Figure 3: h(m, n) = (f (m), g(n))

Como h é sobrejetiva, pelo Corolário 2, basta provar que N×N é enumerável para con-
cluirmos que X ×Y seja, também, enumerável. Para isso, tomemos a função i : N×N → N
dada por i(m, n) = 3n · 5m , pela unicidade da decomposição em fatore primos, temos que
i é injetiva. E como N é enumerável, pelo corolário 1, temos que N × N é enumerável. 

Corolário 4: Uma famı́lia de conjuntos enumeráveis é enumerável.

Demonstração
Tomemos conjuntos enumeráveis X1 , X2 , X3 , · · · Xn , · · · , logo isso implica que existem
bijeções f1 : N → X1 , f2 : N → X2 , f3 : N → X3 , · · · , fn : N → Xn , · · · , e consequente-

[
mente as funções f1 , f2 , f3 , · · · , fn , · · · são sobrejetivas. Chamemos X = Xn .
n=1
Note que a função g : N × N → X definida por g(m, n) = fn (m) é sobrejetiva. De
fato, como as funções fi , com i = 1, 2, 3, ..., são sobrejetivas, temos que ∀fn (m) ∈ X ⇒
m ∈ N e n ∈ N ⇒ (m, n) ∈ N × N, ou seja, ∀fn (m) ∈ X, ∃(m, n) ∈ N × N tal que
g(m, n) = fn (m), e como N × N é enumerável, temos pelo Corolário 2, que X, também, é
enumerável.
Se a reunião for finita, então basta fixa o ı́ndice n fazendo o conjunto I = {1, 2, 3, · · · , n}
daı́ restringindo a função g para g : I × N → X. 
Baseando-nos no que deduzimos nessa seção, temos que na seção 1.3 podemos reescr-
ever o Teorema 3 da seguinte forma:
Todo conjunto infinito contém um subconjunto infinito enumerável.

Exemplo 1: O conjunto dos números negativos Z∗− = {−1, −2, −3, · · · } é enumerável,
basta definir a função bijetiva f : N → Z∗− pondo f (n) = −n.

Exemplo 2: O conjunto Z = {..., −2, −1, 0, 1, 2, ...} dos números inteiros é enu-
(n − 1)
merável. Uma bijeção f : N → Z pode ser definida pondo f (n) = para n ı́mpar
2
9
−n
e f (n) = para n par.
2
m
 
Exemplo 3: O conjunto Q = ; m, n ∈ Z, com n 6= 0 dos números racionais é
n
enumerável. Com efeito, escrevendo Z∗ = Z − {0}, podemos definir uma função sobreje-
m
tiva f : Z × Z∗ → Q pondo f (m, n) = .
n
Exemplo 4: (Um conjunto não-enumerável.)
Seja S o conjunto de todas as sequências infinitas da forma s = (0, 1, 1, 0, 0, 0, 1, 0, · · · ),
onde os elementos em cada sequência são somente o 0 ou o 1.
Suponhamos, absurdamente, que o conjunto S fosse enumerável. Então poderı́amos
fazer uma contagem das sequências do conjunto S como primeira (s1 ), segunda (s2 ),
terceira (s3 ) sequências e assim por diante, independente de quem seja as sequências
escolhidas como primeira, segunda, terceira, etc; o importante é contar. Pode ser, por
exemplo, a contagem a seguir

s1 = (1, 1, 0, 0, · · · )
s2 = (0, 0, 1, 0, · · · )
s3 = (1, 1, 1, 1, · · · )
..
.
sn = (0, 1, 0, 0, · · · , 0, · · · )
..
.

Observe que o 1o elemento de s1 é 1, mas iremos trocar por 0, o 2o elemento de s2


é 0, mas iremos trocar por 1, o 3o elemento de s3 é 1, mas iremos trocar por 0, · · · ,
o n-ésimo elemento de sn é 0, mas iremos trocar por 1, e fazer essa lógica em todas as
sequências dessa contagem. Depois com esses novos elementos vermelhos encontrados
nessa troca façamos uma nova sequência sx com os seus elementos obedecendo suas re-
spectivas posições como primeiro, segundo, terceiro, ..., n-ésimo elemento, e assim por
diante, da seguinte forma

sx = (0, 1, 0, · · · , 1, · · · )
Observe que nessa construção a sequência sx 6= s1 pelo menos no 1o termo, sx 6= s2
pelo menos no 2o termo, sx 6= s3 pelo menos no 3o termo, etc, sx 6= sn pelo menos no
n-ésimo termo, e assim por diante. Ou seja, sx 6= sn , ∀n ∈ N. E isso é um absurdo, pois
sx é uma sequência que não está em nossa contagem.
Logo, o conjunto S é não-enumerável. 

(Este raciocı́nio, devido a G. Cantor, é conhecido como “método da diagonal”.)

10
1.5 Exercı́cios
Seção 1: Números naturais

1) Usando indução, prove:


(a) 1 + 2 + · · · + n = n(n + 1)/2;
(b) 1 + 3 + 5 + · · · + (2n − 1) = n2 .

2) Dados m, n ∈ N com n > m, prove que ou n é múltiplo de m ou existem q, r ∈ N tais


que n = mq + r com r < m. Prove que q e r são únicos com esta propriedade.

3) Seja X ⊂ N um subconjunto não-vazio tal que m, n ∈ X ⇔ m + n ∈ X. Prove que


existe q ∈ N tal que X é o conjunto dos múltiplos de q.

4) Dado n ∈ N, prove que não existe x ∈ N tal que n < x < n + 1.

5) Prove o princı́pio de indução como uma consequência do princı́pio da boa ordenação.

Seção 2: Conjuntos finitos

1) Indicando com card X o número de elementos do conjunto finito X, prove:


(a) Se X é finito e Y ⊂ X então card Y ≤ card X.
(b) Se X e Y são finitos então X ∪ Y é finito e card(X ∪ Y ) = card X + card Y −
T
card(X Y ).
(c) Se X e Y são finitos então X × Y é finito e card(X × Y ) = card X · card Y .

2) Seja P (X) o conjunto cujos elementos são os subconjuntos de X. Prove por indução
que se X é finito então card P (X) = 2card X

3) Seja F (X; Y ) o conjunto das funções f : X → Y . Se card X = m e card Y = n, prove


que card F (X; Y ) = nm .

4) Prove que todo conjunto finito não-vazio X de números naturais contém um elemento
máximo (isto é, existe x0 ∈ X tal que x ≤ x0 , ∀x ∈ X).

Seção 3: Conjuntos infinitos

1) Dada f : X → Y , prove:
(a) Se X é infinito e f é injetiva então Y é infinito.
(b) Se Y é infinito e f é sobrejetiva, então X é infinito.

2) Sejam X um conjunto finito e Y um conjunto infinito. Prove que existe uma função
injetiva f : X → Y e uma função sobrejetiva g : Y → X.

3) Prove que o conjunto P dos números primos é infinito.

4) Dê exemplo de uma sequência decrescente X1 ⊃ X2 ⊃ · · · ⊃ Xn ⊃ · · · de conjuntos


infinitos cuja interseção Xn∞ seja vazia.
T

11
Seção 4: Conjuntos enumeráveis
1) Defina f : N × N → N pondo f (1, n) = 2n − 1 e f (m + 1, n) = 2m (2n − 1). Prove que
f é uma bijeção.
2) Prove que existe g : N → N sobrejetiva tal que g −1 (n) é infinito, para cada n ∈ N.
3) Exprima N = N1 ∪ N2 ∪ · · · ∪ Nn ∪ · · · como união infinita de subconjuntos infinitos,
dois a dois disjuntos.
4) Para cada n ∈ N , seja Pn = {X ⊂ N; card X = n}. Prove que Pn é enumerável.
Conclua que o conjunto Pf dos subconjuntos finitos de N é enumerável.
5) Prove que o conjunto P (N) de todos os subconjuntos de N não é enumerável.
6) Sejam Y enumerável e f : X → Y tal que, para cada y ∈ Y , f −1 (y) é enumerável.
Prove que X é enumerável.

12
1.6 Soluções
Seção 1: Números naturais

1) Usando indução, prove:

(a) 1 + 2 + · · · + n = n(n + 1)/2;


Chamemos 1 + 2 + · · · + n = n(n + 1)/2 de propriedade P. Caso n = 1 temos
1 = 1(1 + 1)/2 = 2/2 = 1 que é verdade.
Suponhamos, por hipótese de indução, que a propriedade P seja válida para
algum n > 1, agora, somando em ambos os membros de P, dessa validade, o
termo (n + 1) obtemos

1 + 2 + · · · + n + (n + 1) = n(n + 1)/2 + (n + 1)

1 + 2 + · · · + n + (n + 1) = [n(n + 1) + 2(n + 1)]/2

1 + 2 + · · · + n + (n + 1) = [(n + 1)(n + 2)]/2

1 + 2 + · · · + n + (n + 1) = {(n + 1)[(n + 1) + 1]}/2

Logo, por indução, P é válida, ∀n ∈ N.


(b) 1 + 3 + 5 + · · · + (2n − 1) = n2 .
Chamemos 1 + 3 + 5 + · · · + (2n − 1) = n2 de propriedade P. Caso n = 1 temos
1 = 12 = 1 que é verdade.
Suponhamos, por hipótese de indução, que a propriedade P seja válida para
algum n > 1, agora, somando em ambos os membros de P, dessa validade, o
termo (2n + 1) obtemos

1 + 3 + 5 + · · · + (2n − 1) + (2n + 1) = n2 + (2n + 1)

1 + 3 + 5 + · · · + (2n − 1) + (2n + 1) = (n + 1)2

Portanto, por indução, temos que a propriedade P é válida ∀n ∈ N.

2) Dados m, n ∈ N com n > m, prove que ou n é múltiplo de m ou existem q, r ∈ N tais


que n = mq + r com r < m. Prove que q e r são únicos com esta propriedade.
Provemos, inicialmente, que dados m, n ∈ N com n > m, então existe q ∈ N tal que
qm ≤ n < (q + 1)m.
Suponha, por absurdo, que não exista esse q. Disso temos duas opções
1a opção: ∀q ∈ N terı́amos qm < (q + 1)m < n tornando a sequência dos múltiplos
de m limitada por n. E isso implicaria que a sequência dos múltiplos de m seria
finita, o que é um absurdo.

13
2a opção: ∀q ∈ N terı́amos n < qm < (q +1)m, daı́ para q = 1 terı́amos n < m < 2m
o que é um absurdo, pois m < n.
Portanto, existe q ∈ N tal que qm ≤ n < (q + 1)m ou qm < n ≤ (q + 1)m.
Isso implica que ou qm = n [ou (q + 1)m = n] ou qm < n < (q + 1)m ⇒ ∃r ∈
N tal que n = qm + r < (q + 1)m ⇒ qm + r < qm + m ⇒ r < m. Ou seja,
n = qm + r com r < m.
Suponha,por absurdo, que q não é único com essa propriedade, mas que r seja
único, ou seja ∃q1 ∈ N com q1 6= q tal que n = q1 m + r. Daı́, qm + r = q1 m + r,
pela lei do corte, terı́amos q = q1 absurdo. Caso q fosse único e r não fosse único
então ∃r1 ∈ N com r1 6= r satisfazendo que qm + r = qm + r1 , pela lei do corte,
terı́amos r = r1 , absurdo. Agora, se ambos, q e r não fossem únicos então ∃q1 , r1 ∈
N com q1 6= q e r1 6= r, digamos r > r1 , satisfazendo que qm + r = q1 m + r1 .
Isso implica que r − r1 = q1 m − qm ⇒ r − r1 = (q1 − q)m ⇒ q1 − q > 0, daı́
r = (q1 − q)m + r1 ⇒ r > (q1 − q)m ⇒ r > m, o que é outro absurdo.

3) Seja X ⊂ N um subconjunto não-vazio tal que m, n ∈ X ⇔ m + n ∈ X. Prove que


existe k ∈ N tal que X é o conjunto dos múltiplos de k.

Denotemos o conjunto dos múltiplos de k como A = {nk; ∀ n ∈ N}.


Para provarmos o que a questão pede, devemos mostrar que A ⊂ X e que X ⊂ A
concluindo assim que A = X.
1o provemos que A ⊂ X
Como X ⊂ N e X 6= ∅, isso implica, pelo P.B.O, que X possui um menor ele-
mento, chamemo-o de k = min{X}, daqui já temos que k ∈ X. Pela propriedade
na ida (⇒) do conjunto X temos que k, k ∈ X ⇒ k + k ∈ X ⇒ 2k ∈ X. Como
k, 2k ∈ X ⇒ 3k ∈ X, suponha, por hipótese de indução, que algum k > 1 satisfaça
que k, nk ∈ X, isso implica que (n + 1)k ∈ X, ou seja, por indução, ∀n ∈ N temos
que nk ∈ X ⇒ A ⊂ X.

2o provemos que X ⊂ A
Note que ∀m ∈ X, com m 6= k, temos k < m. Pelo exercı́cio 2, existem q, r ∈ N
tais que ou m = qk ou que m = qk + r com r < k. Ou seja, somente uma das
afirmações, m = qk e m = qk + r com r < k, é verdadeira.
Mas se existem q, r ∈ N tais que m = qk + r com r < k, temos pela volta (⇐) na
propriedade do conjunto X que qk + r ∈ X ⇒ qk ∈ X e r ∈ X. Só que r ∈ X
contraria a minimalidade de k, pois r < k.
Portanto, qk + r ∈
/ X, somente qk ∈ X. Isto é, todo m ∈ X é da forma qk o que
implica X ⊂ A.
Portanto, A ⊂ X e X ⊂ A ⇒ A = X.

4) Dado n ∈ N, prove que não existe x ∈ N tal que n < x < n + 1.


Suponha, por absurdo, que exista um n ∈ N; n < x < n+1, com x ∈ N, isso implica
que ∃p ∈ N tal que x = n + p. Da mesma forma, ∃q ∈ N tal que n + 1 = x + q. Daı́
encontramos

14
(
x= n+p
n+1= x+q

n+x+1=x+n+p+q

1=p+q

p<1eq<1
(absurdo, pois viola o axioma 2 de Peano)

5) Prove o princı́pio de indução como uma consequência do princı́pio da boa ordenação.

O princı́pio da indução diz que se um conjunto X ⊂ N é tal que 1 ∈ X e s(X) ⊂ X


(isto é, n ∈ X ⇒ s(n) ∈ X) então X = N.
E o P.B.O diz que Todo subconjunto não vazio A ⊂ N possui um menor elemento,
isto é, um elemento n0 ∈ A é tal que n0 < n para todo n ∈ A.
Agora, suponhamos por absurdo, que o conjunto X ⊂ N é tal que 1 ∈ X e s(X) ⊂ X,
mas X 6= N.
Como X 6= N isso implica que N − X 6= ∅. Chamemos A = N − X. O P.B.O
assegura que ∃n0 ∈ A é tal que n0 < n para todo n ∈ A. Note que n0 6=
1, implica (pelo axioma 2 de P eano) que n0 é sucessor de algum número natural.
Mas como n0 é o menor elemento de A então n0 − 1 ∈ X, porém pela propriedade
de X ficamos com s(n0 − 1) = n0 ∈ X. Ou seja, n0 ∈ N − X e n0 ∈ X, o que é um
absurdo, pois N − X e X são conjuntos disjuntos.

Seção 2: Conjuntos finitos

1) Indicando com card X o número de elementos do conjunto finito X, prove:

(a) Se X é finito e Y ⊂ X então card Y ≤ card X.

Temos que X finito implica que ∃f : In → X que é bijeção implicando f −1 :


X → In bijeção com n = card X. Como Y ⊂ X isso implica que Y é finito,
daı́ ∃g : Im → Y bijeção com m = card Y . Isso assegura que f −1 ◦ g : Im → In
é injetiva. O que implica m ≤ n. Ou seja, card Y ≤ card X.

15
(b) Se X e Y são finitos então X ∪ Y é finito e card(X ∪ Y ) = card X + card Y −
T
card(X Y ).
T
Inicialmente, consideremos os conjuntos finitos X e Y disjuntos (ou seja, X Y =
∅). Como X e Y são finitos temos que existem f : In → X e g : Im → Y
bijeções, com n = card X e m = card Y . Agora, construamos a função
h : In+m → X ∪ Y fazendo
(
f (a), se 1 ≤ a ≤ n
h(a) = .
g(a − n), se n + 1 ≤ a ≤ m + n
Suponhamos, por absurdo, que a função h não fosse sobrejetiva.
Logo, ∃b ∈ X ∪ Y tal que ∀a ∈ Im+n não terı́amos h(a) = b. Mas, X Y = ∅
T

nos assegura que b ∈ X ∪ Y ⇒ ou b ∈ X ou b ∈ Y . Se b ∈ X então


∃a ∈ In tal que h(a) = f (a) = b. Se b ∈ Y então ∃a ∈ [n + 1, m + n] e
consequentemente ∃(a − n) ∈ Im , tal que h(a) = g(a − n) = b. Ou seja, não
terı́amos h(a) = b e teriamos h(a) = b, absurdo. Portanto, h é sobrejetiva.
Suponhamos, por absurdo, que a função h não fosse injetiva.
Logo, ∃a, b ∈ X ∪ Y com a 6= b tal que h(a) = h(b) = c. Mas, X Y = ∅
T

nos assegura que c ∈ X ∪ Y ⇒ ou c ∈ X ou c ∈ Y . Se c ∈ X e como f é


injetiva, então ∃a ∈ In tal que h(a) = f (a) = c com a 6= d, ∀d ∈ Im+n − {a}.
Se c ∈ Y e como g é injetiva, então ∃a ∈ [n + 1, m + n] e consequentemente
∃(a − n) ∈ Im , tal que h(a) = g(a − n) = c com a 6= d, ∀d ∈ Im+n − {a} e
com (a − n) 6= d, ∀d ∈ Im+n − {(a − n)} . Ou seja, terı́amos que ∃a, b ∈ X ∪ Y
com a 6= b tal que h(a) = h(b) = c e h(a) = c com a 6= d, ∀d ∈ Im+n − {a}.
Absurdo. Portanto, h é injetiva.
E h sendo sobrejetiva e injetiva ela é bijetiva.
Então X ∪ Y é finito com card(X ∪ Y ) = card X + card Y .

Consideremos agora os conjuntos finitos X e Y não necessariamente disjuntos.


Note que (X − Y ) (X Y ) = ∅ e que X = (X − Y ) ∪ (X Y ).
T T T

Pelo que acabamos de provar temos que card X = card (X −Y )+card (X Y ).


T

O que implica card X − card (X Y ) = card (X − Y ) (I).


T

Note, também, que (X ∪ Y ) = (X − Y ) ∪ Y é também uma união disjunta, daı́


card (X ∪ Y ) = card (X − Y ) + card Y (II).
T
Substituindo (I) em (II) temos card (X∪Y ) = card X−card (X Y )+card Y ,
ou seja, card (X ∪ Y ) = card X + card Y − card (X Y ).
T

(c) Se X e Y são finitos então X × Y é finito e card(X × Y ) = card X · card Y .

Temos que (X f inito) ⇒ ∃f : In → X bijeção e (Y f inito) ⇒ ∃g : Im → Y


bijeção. Logo, n = card X e m = card Y .
Definamos a função h : In × Im → X × Y pondo h(n, m) = (xn , ym ). Note que
h é bijeção, pois se, absurdamente, não fosse, então h não seria sobrejetiva, e
isso implicaria que existiria (xp , yq ) tal que não terı́amos h(p, q) = (xp , yq ), ou
seja, @p ∈ In tal que f (p) = xp , o que é um absurdo, de forma igual terı́amos,
também, @q ∈ Im tal que g(q) = yq que é outro absurdo, pois f e g são bijeções.
Logo, h é sobrejetiva.

16
Se h não fosse injetiva então existiriam (p, q) e (r, s) ∈ In ×Im com (p, q) 6= (r, s)
tal que (xp , yq ) = (xr , ys ), ou seja, xp = xr e yq = ys , mas isso implica, pela
bijetividade de f e g, que p = r e q = s, um absurdo. Logo, h é injetiva. Logo,
de fato, h é bijeção.
Agora, tomemos a função d : In × Im → D definida por d(n, m) = 2n−1 · 3m−1 ,
onde D é o conjunto que contém todos os divisores naturais de 2n−1 · 3m−1 ,
pela unicidade da decomposição em fatores primos temos que d é uma bijeção,
daı́ d−1 : D → In × Im também é bijeção.
Como a quantidade de divisores naturais de 2n−1 · 3m−1 é dada por n · m,
podemos fazer a bijeção j : In·m → D.
Finalmente, façamos a composta h ◦ d−1 ◦ j : In·m → X × Y que é uma bijeção.
Ou seja, X × Y é finito e card(X × Y ) = n · m = card X · card Y .

2) Seja P (X) o conjunto cujos elementos são os subconjuntos de X. Prove por indução
que se X é finito então card P (X) = 2card X

Se X é um conjunto com 1 elemento, digamos X = {e1 } então card X = 1 e


P (X) = {∅, e1 }, logo card P (X) = 2 = 21 o que é verdade.
Suponha, por hipótese de indução, que card P (X) = 2card X
é válida para algum
card X > 1.
Agora, acrescentemos a como mais um elemento de X e com a diferente de todos
os elementos iniciais de X em nossa hipótese. Assim a nova quantidade de ele-
mentos de X fica (card X) + 1. Observe que inicialmente, por hipótese, tı́nhamos
2card X subconjuntos de X. ao acrescentar o elemento a ao subconjunto ∅ ficamos
com o conjunto unitário {a}, suponhamos que e1 seja o primeira elemento de X
então ao acrescentar a ao conjunto unitário {e1 } ficamos com o novo subconjunto
{e1 , a}, fazendo esse processo com todos os subconjuntos 2card X da nossa hipótese,
ficamos com mais 2card X subconjuntos novos. Daı́, o nosso novo conjunto X + {a}
de (card X) + 1 elementos fica com 2card X + 2card X = 2 · 2card X = 2card X+1 sub-
conjuntos ao todo, ou seja, card P (X + {a}) = 2card X+1
Logo, por indução, X sendo finito temos que card P (X) = 2card X
válida ∀ card X ≥
1.
3) Seja F (X; Y ) o conjunto das funções f : X → Y . Se card X = m e card Y = n, prove
que card F (X; Y ) = nm .

Demonstração 1
Dados m, n ∈ N temos que todo elemento da imagem Y pode ser imagem de cada
elemento dos m elementos do domı́nio X. Isso nos assegura a utilização do princı́pio
multiplicativa sem a perda da ideia de função. Então, usando o princı́pio multiplica-
tivo temos que a quantidade de funções f : X → Y é

|n · n ·{zn · · · n}


nm

17
Ilustração

Figure 4: card F (X; Y ) = nm

Demonstração 2
Dados n, m ∈ N, fixemos n e façamos a indução sobre m.
Se m = 1 temos n = n1 funções, o que é verdade.
Suponhamos, por hipótese de indução, que nm seja a quantidade de funções f :
X → Y obedecendo os dados da questão para algum m > 1.
Agora, acrescentemos um elemento xm+1 em X sendo xm+1 diferente de todos os
elementos iniciais de X em nossa hipótese. Daı́, para cada função das nm funções
da hipótese será acrescentada mais n novas funções fazendo a correspondência de
xm+1 para cada um dos n elementos de Y . Resultando assim em n · nm = nm+1
funções ao todo.
Logo, por indução, card F (X; Y ) = nm ; ∀n, m ∈ N, pois m foi induzido e n foi
dado aleatoriamente.

4) Prove que todo conjunto finito não-vazio X de números naturais contém um elemento
máximo (isto é, existe x0 ∈ X tal que x ≤ x0 , ∀x ∈ X).

Definamos o conjunto A = {a ∈ N tal que a > x; ∀x ∈ X}.


Temos X 6= ∅ ⇒ A 6= ∅ o que implica, pelo P.B.O, que ∃m = min{A} (menor
elemento de A) ⇒ m − 1 ∈ / A ⇒ m − 1 ∈ X, pois se m − 1 ∈ / X então todo x ∈ X
estaria no intervalo m − 1 < x < m o que implica X = ∅, absurdo. Agora, se
existisse algum x ∈ X tal que m − 1 < x, então terı́amos m − 1 < x < m, absurdo.
Logo, m − 1 ∈ X e é o maior elemento de X.

18
Seção 3: Conjuntos infinitos

1) Dada f : X → Y , prove:

(a) Se X é infinito e f é injetiva então Y é infinito.

Suponha, por absurdo, que X é infinito, f é injetiva, mas Y fosse finito. Como
Y é finito isso implica que ∃g : In → Y bijeção, o que implica g −1 : Y → In
bijeção. Pelo fato de f ser injetiva temos que a composta g −1 ◦ f : X → In é
injetiva, o que implica X ser finito, absurdo.

(b) Se Y é infinito e f é sobrejetiva, então X é infinito.

f sobrejetiva nos assegura a existência de uma função injetiva g : Y → X dada


por g(f (x)) = x, ∀f (x) ∈ Y . Pelo o que acabamos de provar em (a) temos
que X é infinito.

2) Sejam X um conjunto finito e Y um conjunto infinito. Prove que existe uma função
injetiva f : X → Y e uma função sobrejetiva g : Y → X.

X finito ⇒ ∃f : In → X bijeção ⇒ f −1 : X → In bijeção. E Y infinito ⇒ ∃h :


N → Y injetiva, restringindo h ao domı́nio In temos a função injetiva hr : In → Y .
Fazendo a composta de hr com f −1 temos a função injetiva hr ◦ f −1 : X → Y .
Pelo o que acabamos de provar podemos concluir que hr ◦ f −1 : X → hr (f −1 (X)) é
−1
bijeção com hr (f −1 (X)) ⊂ Y o que implica [hr ◦ f −1 ] : hr (f −1 (X)) → X bijeção.
Como ( Y é infinito podemos construamos a função sobrejetiva g : Y → X pondo
−1 −1
[hr ◦ f ] , se y ∈ hr (f −1 (X))
g=
xc , se y ∈ Y − hr (f −1 (X)) com xc ∈ X

3) Prove que o conjunto P dos números primos é infinito.

Suponhamos, por absurdo, que P seja finito.


Assim, podemos estabelecer uma lista crescente finita dos números primos, digamos
que essa lista seja P = {2 < 3 < 5 < · · · < px < · · · < pn } com n primos.
Essa informação nos assegura que qualquer número natural maior do que pn será
composto. Logo, 2 · 3 · 5 · ... · px · ... · pn + 1 é divisı́vel por algum dos n primos.
Sem perda de generalidade, digamos que ele seja divisı́vel por px , consequentemente
2 · 3 · 5 · ... · px · ... · pn + 1 é o próximo múltiplo de px maior do que 2 · 3 · 5 · ... · px · ... · pn .
Ou seja, terı́amos a igualdade 2 · 3 · 5 · ... · px · ... · pn + 1 = 2 · 3 · 5 · ... · px · ... · pn + px
o que implica 1 = px que é absurdo.
Portanto P é infinito.

19
4) Dê exemplo de uma sequência decrescente X1 ⊃ X2 ⊃ · · · ⊃ Xn ⊃ · · · de conjuntos
infinitos cuja interseção Xn∞ seja vazia.
T

Tomemos
X1 = {1, 2, 3, · · · }
X2 = {2, 3, 4, · · · }
X3 = {3, 4, 5, · · · }
..
.
Xn = {n, n + 1, n + 2, · · · }
..
.
Note que X1 ⊃ X2 ⊃ · · · ⊃ Xn ⊃ · · · . Observe, também, que 1 ∈ X1 , mas 1 ∈
/ X2 .
Isso acontece ∀a ∈ N, isto é, a ∈ Xa , mas a ∈
/ Xa+1 .
Xn∞ = ∅.
T
Agora, mostremos que
Suponhamos, por absurdo, que Xn∞ 6= ∅. Isso implica que existe pelo menos um
T

a ∈ Xn∞ . Mas, pela propriedade de nossa sequência, temos que a ∈ Xa , mas


T

a∈/ Xa+1 , ou seja, a pertence a interseção de todas as sequências ( Xn∞ ), mas não
T

pertence a sequência Xa+1 , o que é um absurdo.


Xn∞ = ∅.
T
Logo, de fato,

Seção 4: Conjuntos enumeráveis

1) Defina f : N × N → N pondo f (1, n) = 2n − 1 e f (m + 1, n) = 2m (2n − 1). Prove que


f é uma bijeção.

Para provarmos que f é injetiva, tomemos os pares (1, n1 ), (1, n2 ) ∈ N × N tais que
(1, n1 ) 6= (1, n2 ), isso implica que n1 6= n2 ⇒ 2n1 6= 2n2 ⇒ 2n1 − 1 6= 2n2 − 1.
Tomando (m1 + 1, n), (m2 + 1, n) ∈ N × N tais que (m1 + 1, n) 6= (m2 + 1, n), isso
implica que m1 +1 6= m2 +1 ⇒ m1 6= m2 ⇒ 2m1 6= 2m2 ⇒ 2m1 (2n−1) 6= 2m2 (2n−1).
Tomando (m + 1, n1 ), (m + 1, n2 ) ∈ N × N tais que (m + 1, n1 ) 6= (m + 1, n2 ), isso
implica que n1 6= n2 ⇒ 2n1 6= 2n2 ⇒ 2n1 −1 6= 2n2 −1 ⇒ 2m (2n1 −1) 6= 2m (2n2 −1).
Tomando (m1 + 1, n1 ), (m2 + 1, n2 ) ∈ N × N tais que (m1 + 1, n1 ) 6= (m2 + 1, n2 ),
isso implica que m1 + 1 6= m2 + 1 e n1 6= n2 ⇒ 2m1 6= 2m2 e 2n1 − 1 6= 2n2 − 1.
Agora, a unicidade da decomposição em fatores primos nos garante a veracidade de
2m1 (2n1 − 1) 6= 2m2 (2n2 − 1).
Para provarmos que f é sobrejetiva suponhamos, por absurdo, que ∃ (2n0 − 1) ∈ N
tal que f (1, n) 6= 2n0 −1, ∀(1, n) ∈ N×N. Isso implica que 2n−1 6= 2n0 −1, ∀n ∈ N,
isto é, n 6= n0 , ∀n ∈ N, absurdo, pois n0 = n0 .

20
Suponhamos, por absurdo, que ∃ 2m0 (2n0 −1) ∈ N tal que f (m+1, n) 6= 2m0 (2n0 −1),
∀(m+1, n) ∈ N×N. Isso implica que 2m (2n−1) 6= 2m0 (2n0 −1), ∀m, n ∈ N, o que é
um absurdo, pois m = m0 e n = n0 garante a igualdade 2m (2n − 1) = 2m0 (2n0 − 1).
Logo, f é injetiva e sobrejetiva, portanto é uma bijeção.

2) Prove que existe g : N → N sobrejetiva tal que g −1 (n) é infinito, para cada n ∈ N.

O Teorema 4 diz que todo subconjunto X ⊂ N é enumerável. Se P é o conjunto


dos números primos, temos que P ⊂ N, logo P é enumerável, além disso, pelo
exercı́cio 3 da seção 3 sabemos que P é infinito. Digamos que a enumeração de P
seja p1 = 2, p2 = 3, p3 = 5, · · · , pn = n − esimo primo, · · · .
Definamos a função g : N → N pondo k = 1, 2, 3, ... com g(pkn ) = n, ∀n ∈ N e
g(m) = m, ∀m 6= pkn para cada n, k ∈ N, sendo m ∈ N.
Suponha, por absurdo, que g não fosse sobrejetiva. Então existiria um n0 ∈ N tal
que g(n) 6= n0 , ∀n ∈ N, o que é um absurdo, pois pn0 ∈ N e g(pn0 ) = n0 . Logo, g é
sobrejetiva.
Note que para cada n ∈ N podemos ter g −1 (n) = g(pkn ) com k = 1, 2, 3, .... Como as
potências dos números primos são infinitas isso assegura que g −1 (n) são infinitos.

3) Exprima N = N1 ∪ N2 ∪ · · · ∪ Nn ∪ · · · como união infinita de subconjuntos infinitos,


dois a dois disjuntos.

Tome os primos listados em ordem crescente {2 < 3 < 5 < · · · < pn < · · · } e
façamos os infinitos conjuntos infinitos

N2 = {2n , ∀n ∈ N}

N3 = {3n , ∀n ∈ N}
N4 = {5n , ∀n ∈ N}
..
.
Nn = {pnn , ∀n ∈ N}
..
.
Note que N2 ∪ N3 ∪ N4 · · · ∪ Nn ∪ · · · 6= N ⇒ N − N2 ∪ N3 ∪ N4 · · · ∪ Nn ∪ · · · 6= ∅
e infinito. Chamemos N1 = N − N2 ∪ N3 ∪ N4 · · · ∪ Nn ∪ · · · . Portanto, temos que
N = N1 ∪ N2 ∪ · · · ∪ Nn ∪ · · · .

4) Para cada n ∈ N , seja Pn = {X ⊂ N; card X = n}. Prove que Pn é enumerável.


Conclua que o conjunto Pf dos subconjuntos finitos de N é enumerável.

Observe que ∀n ∈ N, cada Pn é um conjunto composto por todos os grupos


{a1 , a2 , a3 , . . . , an }, onde a1 , a2 , a3 , . . . , an ∈ N com a1 6= a2 6= a3 6= · · · =
6 an .
n
Agora, note que Pn = N − {n, n, n, · · · , n}, ∀n ∈ N. Pela unicidade da decom-
posição de números naturais em fatores primos temos que g : Pn → N definida por
g(a1 , a2 , a3 , · · · , an ) = 2a1 · 3a2 · 5a3 · ... · (P RIM O)ann tomados os n primeiros primos
é injetiva. Logo, pelo Corolário 1 do Teorema 4, cada Pn é enumerável. E pelo

21
Corolário 4 do Teorema 4 temos que o conjunto Pf dos subconjuntos finitos de N é
enumerável, pois é uma famı́lia de conjuntos enumeráveis.

5) Prove que o conjunto P (N) de todos os subconjuntos de N não é enumerável.

Provemos, inicialmente, que todo subconjunto de um conjunto infinito enumerável


é enumerável.
Se o subconjunto A de um conjunto infinito enumerável X for finito, já temos que
A é enumerável.
Se o subconjunto A de um conjunto infinito enumerável X for infinito então façamos
da seguinte forma. Suponha, por absurdo, que dado um conjunto infinito enumerável
X existe um subconjunto não enumerável A ⊂ X. Pelo fato de X ser enumerável,
então existe uma bijeção f : N → X ⇒ f −1 : X → N é bijeção. Mas, A não é
enumerável, isso implica que não existe uma bijeção g : N → A, daı́ não podemos
fazer a bijeção composta g ◦ f −1 : X → A, ou seja, não existe uma bijeção de X em
A. Porém, a contra-positiva do Corolário do Teorema 3 implica que X é finito, o
que é um absurdo.
Suponhamos, por absurdo, que P (N) fosse finito. Agora, façamos uma função in-
jetiva f : N → P (N) definida por f (n) = {n} (cada natural tem somente um
correspondente com seu subconjunto unitário), mas, como supomos que P (N) fosse
finito, isso implica, pelo Corolário 1 do Teorema 1, que N seria finito, absurdo.
Logo, P (N) é infinito.
Provemos, agora, que existem infinitos subconjuntos infinitos de N.
Para isso selecione todas as potências de 2 e forme o subconjunto {2, 22 , 23 , · · · },
da mesma forma com 3, ou seja, {3, 32 , 33 , · · · }, façamos isso com o 5 também
{5, 52 , 53 , · · · }, assim por diante com todos os números primos. Note que esse pro-
cesso nos dá alguns infinitos subconjuntos infinitos de N, e consequentemente nos
garante que existem infinitos subconjuntos infinitos de N.
Finalmente, suponhamos, por absurdo, que P (N) fosse infinito enumerável. E
tomemos todos os seus infinitos subconjuntos infinitos de números naturais, pelo
que provamos inicialmente, o conjunto X∞ ⊂ P (N) formado por todos esses infini-
tos subconjuntos infinitos de números naturais é enumerável.
Escrevamos os elementos de cada sequência xc ∈ X∞ em ordem crescente.
Agora, obedeçamos a
regra: Contemos posicionalmente os números (um, dois, tres, · · · ) e se os elemen-
tos escritos em ordem crescente de uma sequência xc ∈ X∞ estiverem na contagem
registre 1 na posição da contagem, se não estiverem na contagem registre 0 na
posição da contagem.
Por exemplo, tomemos a sequência x = (1, 3, 7, · · · )
Comece a contar: um (veja que 1 está na posição um da sequência x, então registre
1 no lugar do um da contagem), dois (veja que o dois não está na sequência x então
registre 0 no lugar do dois na contagem, e o registro fica (1, 0)), três (veja que o
três está na sequência x então registre 1 no lugar do três na contagem, e o registro
fica (1, 0, 1)), quatro (veja que o quatro não está na sequência x então registre 0 no

22
lugar do quatro na contagem, e o registro fica (1, 0, 1, 0)), cinco (veja que o cinco
não está na sequência x então registre 0 no lugar do cinco na contagem, e o registro
fica (1, 0, 1, 0, 0)), seis (veja que o seis não está na sequência x então registre 0 no
lugar do seis na contagem, e o registro fica (1, 0, 1, 0, 0, 0)), sete (veja que o sete
está na sequência x então registre 1 no lugar do sete na contagem, e o registro fica
(1, 0, 1, 0, 0, 0, 1), e assim por diante formando uma sequência infinita de zeros e uns
(1, 0, 1, 0, 0, 0, 1, · · · )
Veja que encontramos uma relação que leva x = (1, 3, 7, · · · ) para (1, 0, 1, 0, 0, 0, 1, · · · )

(1, 3, 7, · · · ) → (1, 0, 1, 0, 0, 0, 1, · · · )

Agora, vamos formalizar essa ideia. Denotemos a função r : N → {0, 1} como função
r=regra, definida por (
1, se n ∈ xc
r(n) =
0, se n ∈
/ xc
onde xc = {x1 < x2 < x3 < · · · < xn < · · · } ∈ X∞ e com as imagens r(N) formemos
a sequência infinita de Cantor c = (r(1), r(2), r(3), · · · , r(n), · · · ). Observe, ainda,
que a sequência c0 = (0, 0, 0, . . . ) formada só por zeros não pode ser gerada por r,
pois a função r depende de darmos alguma sequência xc ∈ X∞ .
Com isso podemos definir a função f : X∞ → C − c0 pondo f (xc ) = c em obediência
à função contagem r para cada xc ∈ X∞ , onde C é o conjuntos das sequências
infinitas de Cantor.
Tomemos f (xa ) = f (xb ) = c = (r(1), r(2), r(3), · · · , r(n), · · · ). Isso implica, pela
função r, que xc = xa = xb , logo f é injetiva.
Vamos provar, agora, que f é sobrejetiva.
Suponha, por absurdo, que f não é sobrejetiva. Logo, existe pelo menos uma
sequência c ∈ C que não é imagem de nenhuma sequência de X∞ por f , ou seja,
terı́amos uma sequência infinita de Cantor c que não pode ser gerada pela função
contagem r. Essa sequência não pode possui nenhum dı́gito 1, pois se possuı́sse
então existiria alguma sequência xc ∈ X∞ com n ∈ xc que serviria para o uso da
função r. Consequentemente a sequência (0, 0, 0, . . . ) é a única que não pode ser
gerada por f . Portanto a função f é sobrejetiva.
Portanto, f é injetiva e sobrejetiva, logo a função f : X∞ → C − c0 é uma bijeção.
Mas como X∞ é supostamente enumerável, existe uma bijeção h : N → X∞ , o que
implica que podemos fazer a bijeção composta f ◦ h : N → C − c0 , isto é, C − c0 seria
enumerável, absurdo! Logo, X∞ não é enumerável, como X∞ ⊂ P (N) isso implica
que P (N) não é enumerável.

6) Sejam Y enumerável e f : X → Y tal que, para cada y ∈ Y , f −1 (y) é enumerável.


Prove que X é enumerável.

Note que X = ∪y∈Y (f −1 (y)), portanto é a reunião de conjuntos enumeráveis, logo


X é enumerável.

23
2 Números Reais
O conjunto dos números reais será indicado por R. Faremos neste capı́tulo uma de-
scrição de suas propriedades que, juntamente com suas consequências, serão utilizadas
nos capı́tulos seguintes.

2.1 R é um corpo
Isto significa que estão definidas em R duas operações, chamadas adição e multiplicação,
que cumprem certas condições, abaixo especificadas.
A adição faz corresponder a cada par de elementos x, y ∈ R, sua soma x + y ∈ R,
enquanto a multiplicação associa a esses elementos o seu produto x · y ∈ R.
Os axiomas a que essas operações obedecem são:

Associatividade: para quaisquer x, y, z ∈ R tem-se (x + y) + z = x + (y + z) e (x · y) · z =


x · (y · z).

Comutatividade: para quaisquer x, y ∈ R tem-se x + y = y + x e x · y = y · x.

Elementos neutros: existem em R dois elementos distintos O e 1 tais que x + 0 = x


e x · 1 = x para qualquer x ∈ R.

Inversos: todo x ∈ R possui um inverso aditivo −x ∈ R tal que x + (−x) = 0 e, se


x 6= 0, existe também um inverso multiplicativo x−1 ∈ R tal que x · x−1 = 1.

Distributividade: para x, y, z ∈ R quaisquer, tem-se x · (y + z) = x · y + x · z.

Provemos que x · 0 = 0.
Veja que x · 0 + x = x · 0 + x · 1 = x(0 + 1) = x · 1 = x ⇒ x · 0 + x = x, somando −x
em ambos os membros da última igualdade temos x · 0 + x − x = x − x ⇒ x · 0 = 0.
Provemos que x · y = 0 ⇒ x = 0 ou y = 0.
Se y 6= 0 temos que x · (y.y −1 ) = 0 · y −1 ⇒ x · 1 = 0 ⇒ x = 0. Para y = 0 a prova é
similar.
Provemos, agora, as “regras dos sinais”.
(−x) · y + xẏ = (−x + x)y = 0 · y = 0 ⇒ (−x) · y + xẏ = 0 ⇒ (−x) · y + xẏ − xẏ =
0 − xẏ ⇒ (−x) · y = −xẏ. Para x · (−y) = −xẏ o processo é o mesmo.
Temos que −(−z) + (−z) = 0 ⇒ −(−z) + (−z) + z = 0 + z ⇒ −(−z) = z. Em
particular, −(−1) = 1.
Agora, (−x)(−y) = −x(−y) = −(−xy) = xy.
Provemos, ainda, que se dois números reais x, y têm quadrados iguais então x = ±y.
x2 = y 2 ⇒ x2 − y 2 = 0 ⇒ (x − y)(x + y) = 0 ⇒ x − y = 0 ou x + y = 0 ⇒ x =
y ou x = −y ⇒ x = ±y.

2.2 R é um corpo ordenado


Isto significa que existe um subconjunto R∗+ ⊂ R, (o * exclui o número 0), chamado o
conjunto dos números reais positivos , que cumpre as seguintes condições:

P1) A soma e o produto de números reais positivos são positivos. Ou seja, x, y ∈ R∗+ ⇒
x + y ∈ R∗+ e x · y ∈ R∗+ .

24
P2) Dado x ∈ R, exatamente uma das três alternativas seguintes ocorre: ou x = 0, ou
x ∈ R∗+ ou −x ∈ R∗+ .

Note que caso x ∈ R∗+ então −x ∈R − R∗+ . Então podemos reescrever o conjunto dos
números reais como sendo R = R∗+ ∪ R∗ − R∗+ ∪ {0}.
Provemos que todo número real x 6= 0 tem quadrado positivo.
De fato, caso x ∈ R∗+ isso implica por P1 que x · x = x2 ∈ R∗+ .
Caso x ∈ / R∗+ isso implica por P2 que −x ∈ R∗+ , mas isso implica por P1 que
(−x)(−x) = x2 ∈ R∗+ .
Escreve-se x < y e diz-se que x é menor do que y quando y − x ∈ R∗+ , isto é, y = x + z
onde z é positivo.
Valem as seguintes propriedades da relação de ordem x < y em R:

01. Transitividade: se x < y e y < z então x < z.


Demonstração: x < y e y < z significam y − x ∈ R∗+ e z − y ∈ R∗+ . Por Pl,
segue-se que (y − x) + (z − y) ∈ R∗+ , isto é, z − x ∈ R∗+ , ou seja, x < z. 

02. Tricotomia: dados x, y ∈ R , ocorre exatamente uma das alternativas x = y, x < y


ou y < x.
Demonstração: Dados x, y ∈ R, ou y − x ∈ R∗+ , ou y − x = 0 ou x − y ∈ R∗+ . No
primeiro caso tem-se x < y, no segundo x = y e no terceiro y < x. Estas alternativas
se excluem mutuamente, por P2. 

03. Monotonicidade da adição: se x < y então, para todo z ∈ R, tem-se x + z < y + z.


Demonstração: Se x < y então y − x ∈ R∗+ , donde (y + z) − (x + z) = y − x ∈ R∗+ ,
isto é, x + z < y + z. 

04. Monotonicidade da multiplicação: se x < y então, para todo z > 0 tem-se xz < yz.
Se, porém, z < 0 então x < y implica yz < xz. 
Demonstração: Se x < y e z > 0 então y −x ∈ R∗+ e z ∈ R∗+ , logo (y −x)·z ∈ R∗+ ,
ou seja, yz − xz ∈ R∗+ , o que significa xz < yz. Se x < y e z < 0 então y − x ∈ R∗+
e −z ∈ R∗+ , donde xz − yz = (y − x)(−z) ∈ R∗+ , o que significa yz < xz. 

Mais geralmente, x < y e x0 < y 0 implicam x+x0 < y+y 0 . Com efeito (y+y 0 )−(x+x0 ) =
(y − x) + (y 0 − x0 ) ∈ R∗+ . Analogamente, 0 < x < y e 0 < x0 < y 0 implicam xx0 < yy 0 pois
yy 0 − xx0 = yy 0 − yx0 + yx0 − xx0 = y(y 0 − x0 ) + (y − x)x0 > 0.

Se 0 < x < y então y −1 < x−1 . Para provar, nota-se primeiro que (x−1 )2 > 0 ⇒
x · (x−1 )2 > x · 0 ⇒ x · (x−1 )2 > 0 ⇒ (x · x−1 ) · x−1 > 0 ⇒ 1 · x−1 > 0 ⇒ x−1 > 0.
Da mesma forma, encontramos y −1 > 0, logo por P1 temos x−1 y −1 > 0. Daı́ podemos
multiplicar ambos os membros da desigualdade x < y por x−1 y −1 e encontrar y −1 < x−1 .

Como 1 ∈ R é positivo, segue-se que 1 < 1 + 1 < 1 + 1 + 1 < · · · Podemos então


considerar N ⊂ R. Segue-se que Z ⊂ R pois 0 ∈ R e n ∈ R ⇒ −n ∈ R∗ − R∗+ . Além disso,
m
se m, n ∈ Z com n 6= 0 então = m · n−1 ∈ R, o que nos permite concluir que Q ⊂ R.
n
Assim, N ⊂ Z ⊂ Q ⊂ R.

25
Na seção seguinte, veremos que a inclusão Q ⊂ R é própria.

Exemplo 1. (Desigualdade de Bernoulli.) Para todo número real x ≥ −1 e todo


n ∈ N, tem-se (1 + x)n ≥ 1 + nx.
Isto se prova por indução em n, sendo óbvio para n = 1.
Supondo a desigualdade válida para algum n > 1, multiplicamos ambos os membros
pelo número 1 + x ≥ 0 e obtemos (1 + x)n+1 = (1 + x)n (1 + x) ≥ (1 + nx)(1 + x) =
1 + nx + x + nx2 = 1 + (n + 1)x + nx2 ≥ 1 + (n + 1)x.
Logo, por indução a Desigualdade de Bernoulli é verdadeira ∀n ∈ N. 

A relação de ordem em R permite definir o valor absoluto (ou módulo) de um número


real x ∈ R assim: |x| = x se x > 0, |0| = 0 e |x| = −x se x < 0. Noutras palavras,
|x| = max{x, −x} é o maior dos números reais x e −x.

Exemplo 2. | − 3| = max{3, −3} = 3.

Chamemos o mı́nimo entre dois números a ≤ b como sendo min{a, b} = a.

Exemplo 3. min{−1, 2} = −1.

Tem-se −|x| ≤ x ≤ |x| para todo x ∈ R. Com efeito, a desigualdade x ≤ |x| é óbvia,
enquanto −|x| ≤ x resulta de multiplicar por −1 em ambos os membros da desigualdade
−x ≤ |x|.

Teorema 1. Se x, y ∈ R então |x + y| ≤ |x| + |y| e |x · y| = |x| · |y|.

Demonstração: Somando membro a membro as desigualdades |x| ≥ x e |y| ≥ y vem


|x| + |y| ≥ x + y. Analogamente, de |x| ≥ −x e |y| ≥ −y resulta |x| + |y| ≥ −(x + y).
Logo |x| + |y| ≥ |x + y| = max{x + y, −(x + y)}.
E, |x · y|2 = (x · y)2 = x2 · y 2 = |x|2 · |y|2 = (|x| · |y|)2 ⇒ |x · y| = |x| · |y|. 

Teorema 2. Sejam a, x, δ ∈ R. Tem-se |x − a| < δ se, e somente se, a − δ < x < a + δ.

Demonstração: Como |x − a| é o maior dos dois números x − a e −(x − a), afirmar


que |x − a| < δ equivale a dizer que se tem x − a < δ e −(x − a) < δ, ou seja, x − a < δ
e x − a > −δ. Somando a em ambos os membros das desigualdades anteriores, vem:
|x − a| < δ ⇔ x < a + δ e x > a − δ ⇔ a − δ < x < a + δ. 

Quando a = b, o intervalo fechado [a, b] = [a, a] = [b, b] reduz-se a um único elemento


e chama-se um intervalo degenerado.
E quando a < b, tanto o intervalo fechado [a, b] como o intervalo aberto (a, b) são
chamados não-degenerados.

2.3 R é um corpo ordenado completo


Um conjunto X ⊂ R diz-se limitado superiormente quando existe algum b ∈ R tal que
x ≤ b para todo x ∈ X. Neste caso, diz-se que b é uma cota superior de X. Analogamente,
diz-se que o conjunto X ⊂ R é limitado inferiormente quando existe a ∈ R tal que a ≤ x

26
para todo x ∈ X. O número a chama-se então uma cota inferior de X. Se X é limitado
superior e inferiormente, diz-se que X é um conjunto limitado. Isto significa que X está
contido em algum intervalo limitado [a, b] ou, equivalentemente, que existe k > 0 tal que
x ∈ X ⇒ |x| ≤ k.

Seja X ⊂ R limitado superiormente e não-vazio. Um número b ∈ R chama-se o


supremo do conjunto X quando é a menor das cotas superiores de X. Mais explicitamente,
b é o supremo de X quando cumpre as duas condições:

S1. Para todo x ∈ X, tem-se x ≤ b;

S2. Se c ∈ R é tal que x ≤ c para todo x ∈ X então b ≤ c. A condição S2 admite a


seguinte reformulação:

S2’. Se c < b então existe x ∈ X com c < x.


Com efeito, S2’ diz que nenhum número real menor do que b pode ser cota
superior de X. Às vezes se exprime S2’ assim: para todo  > 0 existe x ∈ X
tal que b −  < x.

Escrevemos b = sup X para indicar que b é o supremo do conjunto X.

Exemplo 4. O conjunto X = {1, 2, 3} tem sup X = 3, pois 1 ≤ 3, 2 ≤ 3 e 3 ≤ 3,


além disso, para todo  > 0, existe 3 ∈ X satisfazendo 3 −  < 3. Ou seja, 3 é o maior
elemento de X e ainda é o supremo de X.

Analogamente, se X ⊂ R é um conjunto não-vazio, limitado inferiormente, um número


real a chama-se o ı́nfimo do conjunto X, e escreve-se a = inf X, quando é a maior das
cotas inferiores de X. Isto equivale às duas afirmações:

I1. Para todo x ∈ X tem-se a ≤ x;

I2. Se c ≤ x para todo x ∈ X então c ≤ a. A condição I2 pode também ser formulada


assim:

I2’. Se a < c então existe x ∈ X tal que x < c. De fato, I2’ diz que nenhum número
maior do que a é cota inferior de X. Equivalentemente: para todo  > 0 existe
x ∈ X tal que x < a + .

Exemplo 5. O conjunto X = {1, 2, 3} tem inf X = 1, pois 1 ≤ 1, 1 ≤ 2 e 1 ≤ 3,


além disso para todo  > 0 existe 1 ∈ X satisfazendo 1 < 1 + . Ou seja, 1 é o menor
elemento de X e ainda é o ı́nfimo de X.

Definamos que se um conjunto X possui a caracterı́stica de que todo subconjunto seu


limitado superiormente possui supremo então X é completo.

Vamos provar mais a frente que o conjunto dos números racionais Q não é completo.

Para isso façamos, inicialmente, algumas observações.


 2
a a
Observe que @ ∈ Q tal que = 2.
b b

27
a
De fato, suponha, por absurdo, que exista uma fração irredutı́vel , satisfazendo que
 2 b
a
= 2. Daı́, a2 = 2b2 ⇒ a2 é PAR ⇒ a é PAR ⇒ a = 2c onde c ∈ N.
b
Daqui temos que a2 = 2b2 ⇒ (2c)2 = 2b2 ⇒ 4c2 = 2b2 ⇒ 2c2 = b2 ⇒ b2 é PAR ⇒ b é
PAR ⇒ b = 2d onde d ∈ N.
a 2c c a
Mas a = 2c e b = 2d implica que = = , logo a fração foi simplificada pelo
b 2d d b
a
fator 2, ou seja, não seria irredutı́vel, absurdo.
b
Note que ∀x ∈ Q temos, pela tricotomia, que ou x2 < 2 ou x2 = 2 ou x2 > 2,
mas acabamos de ver que @ x ∈ Q para x2 = 2. Daı́ podemos concluir que o conjunto
X = {x ∈ Q; x > 0 e x2 < 2} juntamente com o conjunto Y = {y ∈ Q; y > 0 e y 2 > 2}
satisfazem que X ∪ Y = Q∗+ , com X ∩ Y = ∅.

Agora, vamos provar que X = {x ∈ Q; x > 0 e x2 < 2} não possui elemento máximo
e que Y = {y ∈ Q; y > 0 e y 2 > 2} não possui elemento mı́nimo.

Devemos mostrar que ∀x ∈ X vai existir um  > 0 tal que (x + )2 < 2. E daı́ concluir
que X não possui elemento máximo.

Para isso, note que x2 < 2 ⇒ 2 − x2 > 0 e x > 0 ⇒ 2x + 1 > 0. Essa duas informações
2 − x2 2 − x2
nos garante > 0. Daqui podemos tomar 0 <  < 1 tal que >  > 0 e
2x + 1 2x + 1
conseguimos duas conclusões interessantes que são (2x + 1) < 2 − x2 e 2 < .

Então (x + )2 = x2 + 2x + 2 , como 2 <  isso implica que

(x + )2 = x2 + 2x + 2 < x2 + 2x + 

(x + )2 < x2 + (2x + 1)

mas como (2x + 1) < 2 − x2 , então

(x + )2 < x2 + (2x + 1) < x2 + 2 − x2

(x + )2 < 2
Ou seja, X não possui elemento máximo.

Devemos mostrar que ∀y ∈ Y teremos y >  > 0 tal que (y − )2 > 2. E daı́ concluir
que Y não possui elemento mı́nimo.

28
Para isso, note que y 2 > 2 ⇒ y 2 − 2 > 0 e y > 0 ⇒ 2y > 0. Essa duas informações
y2 − 2 y2 − 2
nos garante > 0. Daqui podemos tomar 0 <  < y tal que >  > 0 e
2y 2y
conseguimos a conclusão interessante de 2y < y 2 − 2 ⇒ −2y > −y 2 + 2

Então (y − )2 = y 2 − 2y + 2 > y 2 − 2y > y 2 − y 2 + 2 > 2 ou seja, (y − )2 > 2.

Portanto, Y não possui elemento mı́nimo.

Provemos que o conjunto dos números racionais Q não é completo.

Para isso, basta identificar um subconjunto dos Q que não possui supremo em Q.

Então, vamos provar que X = {x ∈ Q; x > 0 e x2 < 2} ⊂ Q não possui supremo em Q.

Suponhamos, por absurdo, que ∃ sup X = s ∈ Q, como sabemos que X ∪ Y = Q∗+ ,


com X ∩ Y = ∅, onde Y = {y ∈ Q; y > 0 e y 2 > 2}. Isso implica que ou s ∈ X ou s ∈ Y ,
não podemos ter s ∈ X, pois, acabamos de provar que X não possui elemento máximo,
logo, so resta s ∈ Y . Mas, como vimos, Y não possui menor elemento, isso implica que
∃r ∈ Y tal que r < s, e o conjunto X, nos assegura que ∃x ∈ X, daı́ x < r < s, ou seja, s
não seria a menor das cotas superiores de X, absurdo. Isso tudo nos leva a concluir que
s∈/X es∈ / Y , daı́ s ∈
/ Q. Ou seja, s ∈ Q e s ∈
/ Q ao mesmo tempo. Absurdo. Portanto,
X não possui supremo pertencente a Q, logo Q não é completo.

Provemos que todo subconjunto limitado superiormente de R possui supremo, e con-


sequentemente é completo..

A construção de R através dos Cortes de Dedekind em Q nos assegura que todo


subconjunto limitado superiormente de R possui supremo. Logo R é um corpo ordenado
completo. 

Observação: Os Cortes de Dedekind nos dão duas interpretações: Uma que


R ⊂ Q e outra o contrário que Q ⊂ R. A interpretação de R ⊂ Q diz que os cortes
são subconjuntos de Q obedecendo a construção do cordo ordenado e não havendo a ne-
cessidade da definição de supremo. Já a interpretação Q ⊂ R necessita da definição de
supremo.

Nesse nosso trabalho escolhemos a interpretação de Q ⊂ R para usarmos a definição


de supremo nos próximos capı́tulos.

Para mais esclarecimentos sobre esse tema: Os Cortes de Dedekind, veja a nossa
referência [3].

Em seguida veremos algumas consequências da completeza de R.

Teorema 3.

i) O conjunto N ⊂ R dos números naturais não é limitado superiormente;

29
Demonstração: Suponha, por absurdo, que N fosse limitado superiormente.
Como N ⊂ R e estamos supondo que N é limitado superiormente então N pos-
sui supremo. Digamos que o supremo de N seja Sup N = x. Agora, note que existe
n ∈ N tal que x − 1 < n, pelo fato de x ser o supremo de N. Daı́, x < n + 1, e isso
implica que x não seria supremo de N. Ou seja, x seria supremo e ao mesmo tempo
não supremo de N, um absurdo. 
1
 
ii) O ı́nfimo do conjunto X = ; n ∈ N é igual a 0;
n

Demonstração: Note que 0 é uma cota inferior de X. E como provamos, em i),


que N não é limitado superiormente temos que ∀ c > 0, com c ∈ R, vai existir um
1 1
n ∈ N tal que n > , daı́ < c. Ou seja, nenhum número real maior do que 0 pode
c n
ser cota inferior de X, portanto a menor das cotas inferiores de X é mesmo o 0. 

iii) Dados a, b ∈ R∗+ , existe n ∈ N tal que n · a > b.

b
Demonstração: Dados a, b ∈ R∗+ e por i) temos que existe n ∈ N tal que n > .
a
Logo, na > b. 

As propriedades i), ii) e iii) do teorema acima são equivalentes e significam que R é
um corpo arquimediano. Na realidade, iii) é devida ao matemático grego Eudoxo, que
viveu alguns séculos antes de Arquimedes.

Teorema 4. (Intervalos encaixados.) Dada uma sequência decrescente I1 ⊃ I2 ⊃


I3 ⊃ · · · ⊃ In ⊃ · · · de intervalos limitados e fechados In = [an , bn ], existe pelo menos um
número real c tal que c ∈ In para todo n ∈ N.

Demonstração.
Como In = [an , bn ], isso nos implica que I1 ⊃ I2 ⊃ I3 ⊃ · · · ⊃ In ⊃ · · · é equivalente
a a1 ≤ a2 ≤ a3 ≤ · · · ≤ b3 ≤ b2 ≤ b1 . Agora, note que a sequência a1 ≤ a2 ≤ a3 ≤ · · · é
limitada superiormente por bn , ∀n ∈ N , portanto possui supremo. Digamos que c seja
o supremo dessa sequência, logo ∀n ∈ N temos an ≤ c. E, pelo fato, de ∀n ∈ N, bn ser
cota superior da sequência a1 ≤ a2 ≤ a3 ≤ · · · , temos que c ≤ bn , ∀n ∈ N, ou seja,
an ≤ c ≤ bn , ∀n ∈ N. 

Teorema 5. O conjunto dos números reais não é enumerável.


Demonstração. Suponha, por absurdo, que (0, 1) ⊂ R seja enumerável. Logo,
podemos expor todos os números dentro de (0, 1) em uma lista, digamos que a lista é

x1 = 0, a11 a12 a13 ...a1n ...


x2 = 0, a21 a22 a23 ...a2n ...
x3 = 0, a31 a32 a33 ...a3n ...
..
.
xn = 0, an1 an2 an3 ...ann ...
..
.

30
Agora, definamos um número y ∈ (0, 1) da seguinte forma y = b1 b2 b3 ...bn ... tal que
b1 6= a11 , b2 6= a22 , b3 6= a33 , · · · , bn 6= ann , · · · . Ou seja, y 6= x1 pelo menos em a11 , y 6= x2
pelo menos em a22 , y 6= x3 pelo menos em a33 , · · · y 6= xn pelo menos em ann , · · · . Dai,
y 6= xn , ∀n ∈ N. Logo, y ∈ (0, 1) é um número que não se encontra na lista de nossa
contagem de todos os números dentro de (0, 1) o que é um absurdo.
Concluı́mos que (0, 1) ∈ R não é enumerável.
Suponha, por absurdo, que R fosse numerável. Logo, todo subconjunto de R seria
enumerável, inclusive (0, 1), e pelo o que acabamos de provar isso é um absurdo.
Logo, R, também não é enumerável. 

Corolário 1. O conjunto dos números irracionais (R − Q) é não enumerável.

Demonstração. Suponha, por absurdo, que R − Q fosse enumerável, isso implica


que (R − Q) ∪ Q = R seria enumerável, absurdo. 

Corolário 2. Todo intervalo não-degenerado é não-enumerável.

Demonstração. Temos que todo intervalo não-degenerado contém um subconjunto


aberto (a, b) e que a função f : (0, 1) → (a, b) definida por f (x) = (b − a)x + a é bijetiva.
De fato, Tomando x, z ∈ (0, 1) com x 6= z temos que (b − a)x 6= (b − a)z ⇒ (b − a)x + a 6=
(b − a)z + a ⇒ f (x) 6= f (z), logo f é injetiva. Suponha, por absurdo, que exista um
y ∈ (a, b) que não seja imagem de nenhum x ∈ (0, 1) por f . Daı́, y 6= (b − a)x + a para
y−a y−a
todo x ∈ (0, 1), mas, isso implica que y − a 6= (b − a)x ⇒ 6= x, ora ∈ (0, 1)
b−a b−a
e é diferente de todo x ∈ (0, 1), absurdo. Logo, f é sobrejetiva. E como provamos no
Teorema 4 que (0,1) é não enumerável a bijeção f assegura que (a,b) é não-enumerável.
Concluindo assim que todo intervalo não-degenerado é não-enumerável, pois contém um
intervalo (a,b) não-enumerável. 

Teorema 6. Todo intervalo não-degenerado I contém números racionais e irracionais.

Demonstração:
Suponha, por absurdo, que um intervalo não degenerado I dos reais não possua
números irracionais, logo esse intervalo seria enumerável, o que é um absurdo, pois o
conjunto dos números reais é não-enumerável. Daı́, todo intervalo não degenerado dos
reais possui infinitos números irracionais (se fosse finitos números irracionais recairia em
um intervalo enumerável).
Tomemos a e b irracionais em um intervalo não degenerado I. Temos, então, que
m m+1
[a, b] ⊂ I. Escrevamos o conjunto dos reais assim R = m∈Z Im , onde Im =
S
,
n n
1
e < b − a, sendo n ∈ Z fixo.
n
m m+1 m+1 m 1
Agora, note que existe um m ∈ Z tal que <a< , temos que = + ,
n n n n n
1 m+1 m 1 m m+1 m
e como < b − a, obtemos = + < + b − a, isto é, < − a + b.
n n n n n n n
m m m
 
Por outro lado temos que < a ⇒ −a < 0 ⇒ − a + b < b, ou seja, obtemos
n n n
m+1 m m+1 m+1
 
< − a + b < b, então < b. Concluindo assim que ⊂ [a, b] ⊂ I,
n n n n

31
m+1
ou seja, o número racional ⊂ I. 
n

2.4 Exercı́cios
Seção 1: R é um corpo

1. Prove as seguintes unicidades:

(a) Se x + a = x para algum x ∈ R, então a = 0;


(b) Se x · u = x para todo x ∈ R então u = 1;
(c) Se x + y = 0 então y = −x;
(d) Se x · y = 1, então y = x−1 .

a c ad + bc a c ac
2. Dados a, b, c, d ∈ R, se b 6= 0 e d 6= 0 prove que + = e · = .
b d bd b d bd
 −1
a b
3. Se a 6= 0 e b 6= 0 em R, prove que (ab)−1 = a−1 · b−1 e conclua que = .
b a
1 − xn+1
4. Prove que = 1 + x + · · · + xn para todo x 6= 1.
1−x
Seção 2: R é um corpo ordenado

1. Para quaisquer x, y, z ∈ R, prove que |x − z| ≤ |x − y| + |y − z|.

2. Prove que ||x| − |y|| ≤ |x − y| para quaisquer x, y ∈ R.

3. Dados x, y ∈ R, se x2 + y 2 = 0 prove que x = y = 0.


" #
n n(n − 1)
4. Prove por indução que (1 + x) ≥ 1 + nx + · x2 se x ≥ 0.
2

5. Para todo x 6= 0 em R, prove que (1 + x)2n > 1 + 2nx.

6. Prove que |a − b| <  ⇒ |a| < |b| + .


n
(xi + λyi )2 é ≥ 0 para todo
X
7. Use o fato de que o trinômio do segundo grau f (λ) =
i=1
λ ∈ R para provar a desigualdade de Cauchy- Schwarz

n
!2 n
! n
!
x2i yi2
X X X
xi y i ≤
i=1 i=1 i=1

Prove ainda que vale a igualdade se, e somente se, existe λ tal que xi = λyi para
todo i = 1, · · · , n, ou então y1 = · · · = yn = 0.

32
a1 an
8. Se ,··· , pertencem ao intervalo (α, β) e b1 , · · · , bn são positivos, prove que
b1 bn
a1 + · · · + an
pertence a (α, β). Nas mesmas condições, se t1 , · · · , tn ∈ R∗+ , prove
b1 + · · · + bn
t1 a1 + · · · + tn an
que também pertence ao intervalo (α, β) .
t1 b1 + · · · + tn bn
Seção 3: R é um corpo ordenado completo

1. Diz-se que uma função f : X → R é limitada superiormente quando sua imagem


f (X) = {f (x); x ∈ X} é um conjunto limitado superiormente. Então põe-se sup f =
sup{f (x); x ∈ X} . Prove que se f, g : X → R são limitadas superiormente o mesmo
ocorre com a soma f + g : X → R tem-se sup(f + g) ≤ sup f + sup g. Dê um
exemplo com sup(f + g) < sup f + sup g. Enuncie e prove um resultado análogo
para inf .
2. Dadas as funções f, g : X → R∗+ limitadas superiormente, prove que o produto f · g :
X → R∗+ é uma função limitada (superior e inferiormente) com sup(f · g) ≤ sup f ·
sup g e inf (f · g) ≥ inf f · inf g. Dê exemplos onde se tenha < e não =.
3. Nas condições do exercı́cio anterior mostre que sup(f 2 ) = (sup f )2 e inf (f 2 ) =
(inf f )2 .
2 − a2
4. Dados a, b ∈ R∗+ com a2 < 2 < b2 , tome x, y ∈ R∗+ tais que x < 1, x < e
2a + 1
b2 − 2
y< . Prove que (a + x)2 < 2 < (b − y)2 e b − y > 0. Em seguida, considere
2b
o conjunto limitado X = {a ∈ R∗+ ; a2 < 2} e conclua que o número real c = sup X
cumpre c2 = 2.
5. Prove que o conjunto dos polinômios com coeficientes inteiros é enumerável. Um
número real chama-se algébrico quando é raiz de um polinômio com coeficientes
inteiros. Prove que o conjunto dos números algébricos é enumerável. Um número
real chama-se transcendente quando não é algébrico. Prove que existem números
transcendentes.
6. Prove que um conjunto I ⊂ R é um intervalo se, e somente se, a < x < b, a, b ∈ I ⇒
x ∈ I.

2.5 Soluções
Seção 1: R é um corpo

1. Prove as seguintes unicidades:

(a) Se x + a = x para algum x ∈ R, então a = 0;


Demonstração: Pela lei do corte temos que x + a = x + 0 ⇒ a = 0.
(b) Se x · u = x para todo x ∈ R∗ então u = 1;
Demonstração: Como x 6= 0 temos seu inverso x−1 daı́ multiplicando em
ambos os membros encontramos u = 1.

33
(c) Se x + y = 0 então y = −x;
Demonstração: basta adicionar −x a ambos os membros.
(d) Se x · y = 1, então y = x−1 .
Demonstração: Basta multiplicar em ambos os membros x−1 .

a c ad + bc a c ac
2. Dados a, b, c, d ∈ R, se b 6= 0 e d 6= 0 prove que + = e · = .
b d bd b d bd

Demonstração: Temos que b−1 · b = 1 e d−1 · d = 1 daı́ multiplicando membro a


membro obtemos b−1 · b · d−1 · d = 1 · 1, ou seja, (b−1 · d−1 ) · (b · d) = 1. mas sabemos,
também, que (b·d)−1 ·(b·d) = 1, portando (b−1 ·d−1 )·(b·d) = 1 = (b·d)−1 ·(b·d), o que
implica (b−1 ·d−1 )·(b·d) = (b·d)−1 ·(b·d) e pela lei do corte temos (b−1 ·d−1 ) = (b·d)−1 .
a c
Agora, basta fazer · = a · b−1 · c · d−1 = (a · c) · (b−1 · d−1 ), e pelo o que acabamos
b d
a c a c a·c
de provar temos · = (a · c) · (b · d)−1 , ou seja, · = .
b d b d b·d
x y x+y
Temos ainda que + = x · z −1 + y · z −1 = (x + y)z −1 = , com base nisso
z z z
podemos fazer
a c a d c b ad cb ad + cb
+ = · + · = + = .
b d b d d b bd bd bd
 −1
−1 −1 −1 a b
3. Se a 6= 0 e b 6= 0 em R, prove que (ab) =a ·b e conclua que = .
b a

Demonstração: Já provamos no exercı́cios 2 que (ab)−1 = a−1 · b−1 . Provemos,


primeiramente, que (b−1 )−1 = b, de fato, veja que b·b−1 = 1 implica que b é o inverso
 −1
−1 −1 −1 a 1 b
de b , ou seja, b = (b ) . Agora, = (a · b−1 )−1 = a−1 · (b−1 )−1 = · b = .
b a a
1 − xn+1
4. Prove que = 1 + x + · · · + xn para todo x 6= 1.
1−x
n n  
xi (x − 1) = xi+1 − xi = xn+1 −1, e isso implica
X X
Demonstração: Note que
i=0 i=0
n n
i n+1
xi = xn+1 − 1, dividindo, ambos os
X X
que x (x − 1) = x − 1, isto é, (x − 1)
i=0 i=0
n
xn+1 − 1
xi =
X
membros dessa igualdade por (x − 1) temos .
i=0 x−1

Seção 2: R é um corpo ordenado

1. Para quaisquer x, y, z ∈ R, prove que |x − z| ≤ |x − y| + |y − z|.

Demonstração:
Pela desigualdade triangular temos |x − z| = |x − y + y − z| ≤ |x − y| + |y − z|. Ou
seja, |x − z| ≤ |x − y| + |y − z|. 

34
2. Prove que ||x| − |y|| ≤ |x − y| para quaisquer x, y ∈ R.
Demonstração: Temos |x| = |x − z + z| pela desigualdade triangular obtemos
|x| = |x−z+z| ≤ |x−z|+|z| ⇒ |x|−|z| ≤ |x−z|. Note,também, que |z| = |z−x+x|
pela desigualdade triangular obtemos |z| = |z − x + x| ≤ |z − x| + |x| ⇒ |z| − |x| ≤
|z−x| ⇒ |z|−|x| ≤ |x−z|, por esses dois últimos resultados temos ||x|−|z|| ≤ |x−z|.


3. Dados x, y ∈ R, se x2 + y 2 = 0 prove que x = y = 0.

Demonstração: Suponhamos, por absurdo, que x 6= 0 daı́ x2 > 0, o que implica


x2 + y 2 > 0 + y 2 , mas por hipótese x2 + y 2 = 0, o que implica 0 > 0 + y 2 , ou seja,
y 2 < 0, absurdo. Logo, x = 0. Fazendo o mesmo raciocı́nio de demonstração para
y 6= 0 encontraremos, também, o mesmo absurdo, logo y = 0. Portanto, x = y = 0.

" #
n n(n − 1)
4. Prove por indução que (1 + x) ≥ 1 + nx + · x2 se x ≥ 0.
2

Demonstração: Se x = 0 temos 1n ≥ 1 é válido para todo n natural.


Vamos provar que a proposição é válida para x > 0.
Para n = 1 temos 1 + x ≥ 1 + x que é verdadeiro.
Suponha, por hipótese de indução, que a proposição é válida para algum n > 1. Daı́
( " # )
n n(n − 1)
(1 + x) · (1 + x) ≥ 1 + nx + · x2 · (1 + x) = 1 + x + (1 + x)(nx) +
" #
2 " #
n(n − 1) n(n − 1)
(1 + x) · x2 = 1 + (n + 1)x + nx2 + (1 + x) · x2 ≥ 1 + (n +
"
2 # "
2 #
(n + 1)n 2 n+1 (n + 1)n 2
1)x + x , ou seja, (1 + x) ≥ 1 + (n + 1)x + x . Portanto,
2 2
a proposição é válida para todo n natural com x ≥ 0.

5. Para todo x 6= 0 em R, prove que (1 + x)2n > 1 + 2nx.

Demonstração:
Pela Desigualdade de Bernoulli temos que a proposição (1 + x)2n > 1 + 2nx é válida
para todo x ≥ −1 e para todo n natural.
Vamos provar agora que ela é válida para todo x < −1 e todo n natural. De fato,
note que x < −1 ⇒ 2nx < −2n ⇒ 1 + 2nx < 1 − 2n < 0 < (1 + x)2n para todo
x < −1 e todo n natural. 

6. Prove que |a − b| <  ⇒ |a| < |b| + .

Demonstração:
Note |a| = |a − b + b| ≤ |a − b| + |b|, ou seja, |a| − |b| ≤ |a − b| < , portanto
|a| < |b| + .

35
n
(xi + λyi )2 é ≥ 0 para todo
X
7. Use o fato de que o trinômio do segundo grau f (λ) =
i=1
λ ∈ R para provar a desigualdade de Cauchy- Schwarz

n
!2 n
! n
!
x2i yi2
X X X
xi y i ≤
i=1 i=1 i=1

Prove ainda que vale a igualdade se, e somente se, existe λ tal que xi = λyi para
todo i = 1, · · · , n, ou então y1 = · · · = yn = 0.

n n n n
(xi + λyi )2 = (x2i + 2xi λyi + λ2 yi2 ) = x2i + λ2
X X X X
Demonstração: xi y i +
i=1 i=1 i=1 i=1
n n n n
λ2 yi2 = λ2 yi2 + λ2 x2i ≥ 0 ⇒ ∆ ≤ 0. Daı́,
X X X X
xi yi +
i=1 i=1 i=1 i=1
n
!2 n n n
!2 n
! n
!
yi2 x2i ≤ 0 ⇒ x2i yi2 .
X X X X X X
2 xi y i −4 xi y i ≤
i=1 i=1 i=1 i=1 i=1 i=1
Temos, também, que
n
!2 n
! n
! n
!2 n
!2
λyi2 2
yi2 λyi2 2
yi2
X X X X X
xi = λyi ⇔ ≤ (λyi ) ⇔ ≤ λ ⇔
i=1 i=1 i=1 i=1 i=1
n n
yi2 = λ yi2 para todo i = 1, · · · , n ou para y1 = · · · = yn = 0.
X X
λ
i=1 i=1

a1 an
8. Se ,··· , pertencem ao intervalo (α, β) e b1 , · · · , bn são positivos, prove que
b1 bn
a1 + · · · + an
pertence a (α, β). Nas mesmas condições, se t1 , · · · , tn ∈ R∗+ , prove
b1 + · · · + bn
t1 a1 + · · · + tn an
que também pertence ao intervalo (α, β) .
t1 b1 + · · · + tn bn
a1 an
Demonstração: α < < β, · · · , α < < β como b1 , · · · , bn são positivos temos
b1 bn
αb1 < a1 < βb1 , · · · , αbn < an < βbn , daı́ α(b1 + · · · + bn ) < a1 + · · · + an < β(b1 +
a1 + · · · + an
· · · + bn ) ou seja, α < < β. Para a segunda parte da demonstração
b1 + · · · + bn
basta multiplicar cada ti no processo da demonstração inicial.

Seção 3: R é um corpo ordenado completo

1. Diz-se que uma função f : X → R é limitada superiormente quando sua imagem


f (X) = {f (x); x ∈ X} é um conjunto limitado superiormente. Então põe-se
sup f = sup{f (x); x ∈ X} . Prove que se f, g : X → R são limitadas superiormente
o mesmo ocorre com a soma f + g : X → R tem-se sup(f + g) ≤ sup f + sup g.
Dê um exemplo com sup(f + g) < sup f + sup g. Enuncie e prove um resultado
análogo para inf .

Demonstração: Primeiramente, note que a 6= b tais que a, b ∈ X implica que


f (a) + g(b) pode pertencer ou não a (f + g)(X). Se f (a) + f (b) = (f + g)(x),

36
onde x ∈ X então [f (a) + g(b)] ∈ (f + g)(X), mas existe a possibilidade de termos
f (a) + f (b) 6= (f + g)(x) para todo x ∈ X então [f (a) + g(b)] ∈
/ (f + g)(X). Por
isso, obrigatoriamente, ficamos com (f + g)(X) ⊆ [f (X) + g(X)].
Agora, f limitada superiormente implica que existe 1 tal que f (x1 ) ≤ 1 para todo
x1 ∈ X. Analogamente, vai existir um 2 tal que g(x2 ) ≤ 2 para todo x2 ∈ X, e
isso implica que f (x1 ) + g(x2 ) ≤ 1 + 2 para todo x1 , x2 ∈ X. Mas, como temos
(f + g)(X) ⊆ [f (X) + g(X)] ficamos, também, com (f + g)(x) ≤ 1 + 2 para todo
x ∈ X. Ou seja, (f + g)(X) é limitado superiormente.
Note, ainda, que f e g limitadas superiormente implica que f (x1 ) ≤ sup f (X) e
g(x2 ) ≤ sup g(X) daı́ f (x1 ) + g(x2 ) ≤ sup f (X) + sup g(X) para todo x1 , x2 ∈ X,
ou seja, sup f (X) + sup g(X) é cota superior de f (X) + g(X), além disso, para
todos 1 > 0 e 2 > 0 existem x1 , x2 ∈ X tais que sup f (X) < f (x1 ) + 1 e
sup g(X) < g(x2 ) + 2 , somando essas desigualdades membro a membro temos,
sup f (X) + sup g(X) < f (x1 ) + g(x2 ) + 1 + 2 , pela definição de supremo, temos
que sup f (X) + sup g(X) = sup [f (X) + g(X)].
E se A e B são conjuntos limitados superiormente tais que A ⊆ B temos que sup B
é cota superior de A, e como o supremo de A é a menor das cotas superiores, temos
que sup A ≤ sup B.
Finalmente,
(f +g)(X) ⊆ [f (X) + g(X)] ⇒ sup [(f + g)(X)] ≤ sup [f (X) + g(X)] = sup f (X)+
sup g(X). Ou seja, sup [(f + g)(X)] ≤ sup f (X) + sup g(X).

Um exemplo com sup(f + g) < sup f + sup g pode ser f (x) = x e g(x) = −x onde
f, g : [0, 1] → R. Daı́, (f +g)(x) = x−x = 0 ⇒ sup (f +g) = 0 e sup f = 1, sup g = 0
donde sup f + sup g = 1 + 0 = 1 ou seja, sup (f + g) = 0 < 1 = sup f + sup g.

Se f, g : X → R são limitadas inferiormente o mesmo ocorre com a soma f +g : X →


R tem-se inf f +inf g ≤ inf (f +g). Dê um exemplo com inf f +inf g < inf (f +g).

Demonstração: f limitada inferiormente implica que existe 1 tal que 1 ≤ f (x1 )


para todo x1 ∈ X. Analogamente, vai existir um 2 tal que 2 ≤ g(x2 ) para todo
x2 ∈ X, e isso implica que 1 + 2 ≤ f (x1 ) + g(x2 ) para todo x1 , x2 ∈ X. Mas, como
acabamos de ver, temos (f + g)(X) ⊆ [f (X) + g(X)], então 1 + 2 ≤ (f + g)(x)
para todo x ∈ X. Isto é, (f + g)(X) é limitado inferiormente.
Agora, f limitada inferiormente implica que existe 1 tal que 1 ≤ f (x1 ) para todo
x1 ∈ X. Analogamente, vai existir um 2 tal que 2 ≤ g(x2 ) para todo x2 ∈ X, e
isso implica que 1 + 2 ≤ f (x1 ) + g(x2 ) para todo x1 , x2 ∈ X. Mas, como temos
(f + g)(X) ⊆ [f (X) + g(X)] ficamos, também, com 1 + 2 ≤ (f + g)(x) para todo
x ∈ X. Ou seja, (f + g)(X) é limitado inferiormente.
Note, ainda, que f e g limitadas inferiormente implica que inf f (X) ≤ f (x1 ) e
inf g(X) ≤ g(x2 ) daı́ inf f (X) + inf g(X) ≤ f (x1 ) + g(x2 ) para todo x1 , x2 ∈ X,
ou seja, inf f (X) + inf g(X) é cota inferior de f (X) + g(X), além disso, para
todos 1 > 0 e 2 > 0 existem x1 , x2 ∈ X tais que f (x1 ) − 1 < inf f (X) e
g(x2 ) − 2 < inf g(X), somando essas desigualdades membro a membro temos,

37
f (x1 ) + g(x2 ) − (1 + 2 ) < inf f (X) + inf g(X), pela definição de ı́nfimo, temos
que inf f (X) + inf g(X) = inf [f (X) + g(X)].
Observe que se A e B são conjuntos limitados inferiormente tais que A ⊆ B temos
que inf B é cota inferior de A, como o ı́nfimo de A é a maior das cotas inferiores,
temos inf B ≤ inf A.
Então como (f + g)(X) ⊆ [f (X) + g(X)] isso implica que
inf f (X) + inf g(X) = inf [f (X) + g(X)] ≤ inf (f + g)(X),
logo, inf f (X) + inf g(X) ≤ inf (f + g)(X).

Um exemplo com inf f + inf g < inf (f + g) pode ser f (x) = x e g(x) = −x onde
f, g : [0, 1] → R. Daı́, (f + g)(x) = x − x = 0 ⇒ inf (f + g) = 0 e inf f = 0,
inf g = −1 donde inf f + inf g = 0 − 1 = −1 ou seja, inf f + inf g = −1 <
0 = inf (f + g).
2. Dadas as funções f, g : X → R∗+ limitadas superiormente, prove que o produto
f · g : X → R∗+ é uma função limitada (superior e inferiormente) com sup(f · g) ≤
sup f · sup g e inf (f · g) ≥ inf f · inf g. Dê exemplos onde se tenha < e não =.

Demonstração: Como estamos trabalhando com conjunto imagem igual aos reais
sempre positivo e diferente de zero podemos ter para todo x ∈ X a existência de
0 < 1 e 0 < 2 tais que f (X) ≤ 1 e g(X) ≤ 2 o que implica f (X)g(X) ≤ 1 2
caso f e g sejam limitadas superiormente. E como as imagens de f e g são somente
números reais positivos então f (X)g(X) é limitada inferiormente pelo 0 ou qualquer
número real negativo. Ou seja, f · g : X → R∗+ é limitada superior e inferiormente.
Podemos ter a 6= b com a, b ∈ X tais que (f · g)(x) = f (a) · g(b) para todo x ∈ X,
ou pode existir um x ∈ X tal que (f · g)(x) 6= f (a) · g(b), e isso assegura que
(f · g)(X) ⊆ f (X) · g(X).
Como vimos na questão 1 temos
sup (f · g)(X) ≤ sup [f (X) · g(X)] e inf [f (X) · g(X)] ≤ inf (f · g)(X).
Vamos provar que sup [f (X) · g(X)] = sup f (X) · sup g(X). De fato,
f (x1 ) ≤ sup f (X) para todo x1 ∈ X e g(x2 ) ≤ sup g(X) para todo x2 ∈ X, como
estamos trabalhando com contra domı́nio positivo, podemos multiplicar membro a
membro, dai f (x1 )g(x2 ) ≤ sup f (X) · sup g(X) para todo x1 , x2 ∈ X. Ou seja,
temos que sup f (X) · sup g(X) é cota superior de f (X) · g(X).
c
Tomando, c < sup f (X)·sup g(X) temos < sup f (X) então vai existir um
sup g(X)
c c
x1 ∈ X tal que < f (x1 ) o que implica que < sup g(X) daı́ vai existir
sup g(X) f (x1 )
c
um x2 ∈ X tal que < g(x2 ) o que implica c < f (x1 )g(x2 ), ou seja, c não pode
f (x1 )
ser uma cota superior de f (X)·g(X), logo sup [f (X) · g(X)] = sup f (X)·sup g(X).
Portanto, sup (f · g)(X) ≤ sup [f (X) · g(X)] = sup f (X) · sup g(X).
Vamos provar que sup [f (X) · g(X)] = sup f (X) · sup g(X). De fato,
f (x1 ) ≤ sup f (X) para todo x1 ∈ X e g(x2 ) ≤ sup g(X) para todo x2 ∈ X, como
estamos trabalhando com contra domı́nio positivo, podemos multiplicar membro a

38
membro, dai f (x1 )g(x2 ) ≤ sup f (X) · sup g(X) para todo x1 , x2 ∈ X. Ou seja,
temos que sup f (X) · sup g(X) é cota superior de f (X) · g(X).
c
Tomando, c < sup f (X)·sup g(X) temos < sup f (X) então vai existir um
sup g(X)
c c
x1 ∈ X tal que < f (x1 ) o que implica que < sup g(X) daı́ vai existir
sup g(X) f (x1 )
c
um x2 ∈ X tal que < g(x2 ) o que implica c < f (x1 )g(x2 ), ou seja, c não pode
f (x1 )
ser uma cota superior de f (X)·g(X), logo sup [f (X) · g(X)] = sup f (X)·sup g(X).
Portanto, sup (f · g)(X) ≤ sup [f (X) · g(X)] = sup f (X) · sup g(X).

3. Nas condições do exercı́cio anterior mostre que sup(f 2 ) = (sup f )2 e inf (f 2 ) =


(inf f )2 .
2 − a2
4. Dados a, b ∈ R∗+ com a2 < 2 < b2 , tome x, y ∈ R∗+ tais que x < 1, x < e
2a + 1
b2 − 2
y< . Prove que (a + x)2 < 2 < (b − y)2 e b − y > 0. Em seguida, considere
2b
o conjunto limitado X = {a ∈ R∗+ ; a2 < 2} e conclua que o número real c = sup X
cumpre c2 = 2.

5. Prove que o conjunto dos polinômios com coeficientes inteiros é enumerável. Um


número real chama-se algébrico quando é raiz de um polinômio com coeficientes
inteiros. Prove que o conjunto dos números algébricos é enumerável. Um número
real chama-se transcendente quando não é algébrico. Prove que existem números
transcendentes.

6. Prove que um conjunto I ⊂ R é um intervalo se, e somente se, a < x < b, a, b ∈ I ⇒


x ∈ I.

Referências
[1] Lima, Elon Lages, Curso de Análise, vol. 1, 12a. edição, Projeto Euclides, IMPA,
2004.

[2] Soluções dos exercı́cios de Análise do livro Análise real volume 1 de Elon Lages
Lima. Rodrigo Carlos Silva de Lima. rodrigo.uff.math@gmail.com

[3] Construção dos números reais vida costes de Dedekind, Thiago Trindade Pi-
mentel, Dissertação de Mestrado do Programa de Mestrado Profissional em Matemática
em Rede Nacional (PROFMAT)- https://teses.usp.br/teses/disponiveis/55/55136/tde-
18102018-164352/publico/ThiagoTrindadePimentel revisada.pdf

[4] Elementos da Análise. Mirian Buss Gonçalves. Daniel Gonçalves . 2a Edição Flo-
rianópolis, 2012. UFSC.

[5] Conjuntos Enumeráveis e Não-Enumeráveis João Antônio Francisconi Lubanco


Thomé Bacharelado em Matemática - UFPR jolubanco@gmail.com Prof. Dr. Fernando de
Ávila Silva (Orientador)Departamento de Matem´atica - UFPR fernando.avila@ufpr.br -
https://pt.scribd.com/document/401423019/Conjuntos-Enumeraveis-e-Nao-Enumeraveis-
14815

39

Você também pode gostar